Задачи на вероятность с экзаменами


Пройти тестирование по этим заданиям
Вернуться к каталогу заданий

Версия для печати и копирования в MS Word

1

Какова вероятность того, что случайно выбранный телефонный номер оканчивается двумя чётными цифрами?


2

Если шахматист А. играет белыми фигурами, то он выигрывает у шахматиста Б. с вероятностью 0,52. Если А. играет черными, то А. выигрывает у Б. с вероятностью 0,3. Шахматисты А. и Б. играют две партии, причём во второй партии меняют цвет фигур. Найдите вероятность того, что А. выиграет оба раза.


3

На рисунке изображён лабиринт. Паук заползает в лабиринт в точке «Вход». Развернуться и ползти назад паук не может, поэтому на каждом разветвлении паук выбирает один из путей, по которому ещё не полз. Считая, что выбор дальнейшего пути чисто случайный, определите, с какой вероятностью паук придёт к выходу D.


4

Вероятность того, что в случайный момент времени температура тела здорового человека окажется ниже чем 36,8 °С, равна 0,81. Найдите вероятность того, что в случайный момент времени у здорового человека температура окажется 36,8 °С или выше.


5

При изготовлении подшипников диаметром 67 мм вероятность того, что диаметр будет отличаться от заданного не больше, чем на 0,01 мм, равна 0,965. Найдите вероятность того, что случайный подшипник будет иметь диаметр меньше чем 66,99 мм или больше чем 67,01 мм.

Пройти тестирование по этим заданиям

Сборник задач по теории вероятностей

(с решениями)

Разработка предназначена для   учащихся  9–11  классов для подготовки к ОГЭ и ЕГЭ по математике.

УМК любой

Цель: показать решение типовых задач по данной теме, закрепить умение учащихся решать данные задачи, подготовить учеников к сдаче ОГЭ и ЕГЭ

Методические рекомендации по использованию ресурса:                    Работу можно применить:

  • при проведении урока по систематизации и закреплении знаний учащихся
  • при проведении консультаций.

Источники информации:                                                       Открытый банк ЕГЭ ФИПИ  http://fipi.ru/                                                       

Теория вероятностей

Классическое определение вероятности
Вероятностью события A называется отношение числа благоприятных для A исходов к числу всех равновозможных исходов:
Р (А) =  
где 
n — общее число равновозможных исходов, m — число исходов, благоприятствующих событию A.
Противоположные события
Событие, противоположное событию A, обозначают Ā. При проведении испытания всегда происходит ровно одно из двух противоположных событий и  
   Объединение несовместных событий
Два события A и B называют несовместными, если отсутствуют исходы, благоприятствующие одновременно как событию A, так и событию B.
Если события A и B несовместны, то вероятность их объединения равна
сумме вероятностей событий  A и B:               P(A U B) =P(A) + P(B)                          

Пересечение независимых событий
Два события A и B называют независимыми, если вероятность каждого из них
не зависит от появления или непоявления другого события.
Событие C называют пересечением событий A и B                     (пишут C = A∩B), если событие C означает, что
произошли оба события A и B.
Если события
A и B независимы, то вероятность их пересечения равна произведению вероятностей  событий A и B:
 P(A∩B) = P(A) • P(B)

Формула сложения вероятностей совместных событий:

                           P(A U B) =P(A) + P(B)   P(A∩B)

1. Из 1000 собранных на заводе телевизоров 5 штук бракованных. Эксперт проверяет один наугад выбранный телевизор из этой 1000. Найдите вероятность того, что проверяемый телевизор окажется бракованным.
 Решение.  При выборе телевизора наугад возможны 1000 исходов, событию A «выбранный телевизор — бракованный» благоприятны 5 исходов. По определению вероятности                       P(A) = 5÷1000 = 0,005. Ответ: 0,005.

2. В урне 9 красных, 6 жёлтых и 5 зелёных шаров. Из урны наугад достают один шар. Какова вероятность того, что этот шар окажется жёлтым?                                                                       Решение. Общее число исходов равно числу шаров: 9 + 6 + 5 = 20. Число исходов, благоприятствующих данному событию, равно 6. Искомая вероятность равна 6÷20 = 0,3.  Ответ: 0,3.

 3. Петя, Вика, Катя, Игорь, Антон, Полина бросили жребий — кому начинать игру. Найдите вероятность того, что начинать игру должен будет мальчик.

Решение. Вероятность события равна отношению количества благоприятных случаев к количеству всех случаев. Благоприятными случаями являются 3 случая, когда игру начинает Петя, Игорь или Антон, а количество всех случаев 6. Поэтому искомое отношение равно 3:6=0,5. Ответ: 0,5.

4. В чемпионате мира участвуют 16 команд. С помощью жребия их нужно разделить на четыре группы по четыре команды в каждой. В ящике вперемешку лежат карточки с номерами групп: 1, 1, 1, 1, 2, 2, 2, 2, 3, 3, 3, 3, 4, 4, 4, 4. Капитаны команд тянут по одной карточке. Какова вероятность того, что команда России окажется во второй группе?

Решение: Обозначим через А событие «команда России во второй группе». Тогда количество благоприятных событий m  = 4 (четыре карточки с номером 2), а общее число равновозможных событий n = 16 (16 карточек) по определению вероятности         Р= 4: 16 = 0,25. Ответ:0,25

5.  В лыжных гонках участвуют 11 спортсменов из России, 6 спортсменов из Норвегии и 3 спортсмена из Швеции. Порядок, в котором спортсмены стартуют, определяется жребием. Найдите вероятность того, что первым будет стартовать спортсмен не из России.

Решение. Всего спортсменов 11 + 6 + 3 = 20 человек. Поэтому вероятность того, что первым будет стартовать спортсмен не из России равна 9:20 = 0,45. Ответ: 0,45.

 6. На каждые 1000 электрических лампочек приходится 5 бракованных. Какова вероятность купить исправную лампочку? 

Решение. На каждые 1000 лампочек приходится 5 бракованных, всего их 1005. Вероятность купить исправную лампочку будет равна доле исправных лампочек на каждые 1005 лампочек, то есть  1000:1005=0,995.Ответ: 0,995.

7. В группе туристов 8 человек. С помощью жребия они выбирают шестерых человек, которые должны идти в село в магазин за продуктами. Какова вероятность того, что турист Д., входящий в состав группы, пойдёт  в магазин? 6 : 8=0,75.

8. В чемпионате по футболу участвуют 16 команд, которые жеребьевкой распределяются на 4 группы: A, B, C и D. Какова вероятность того, что команда России не попадает в группу A?

Решение. Каждая команда попадет в группу с вероятностью 0,25. Таким образом, вероятность того, что команда не   попадает в группу равна 1-0,25=0,75. Ответ:0,75

9. На турнир по шахматам прибыло 26 участников в том числе Коля и Толя. Для проведения жеребьевки первого тура участников случайным образом разбили на две группы по 13 человек. Найти вероятность того, что Коля и Толя попадут в разные группы.                                                                                               Решение. Всего 26 мест. Пусть Коля займет случайное место в любой группе. Останется 25 мест, из них в другой группе 13. Исходом считаем выбор места для Толи. Благоприятных исходов 13. Р=13/25 = 0,52. Ответ:0,52                                                                     

10. В классе 16 учащихся, среди них два друга —Вадим и Сергей. Учащихся случайным образом разбивают на 4 равные группы. Найдите вероятность того, что Вадим и Сергей окажутся в одной группе.                                                                    Решение. Если Сергею первому досталось некоторое место, то Олегу остаётся 15 мест. Из них 3 — в той же группе, где Сергей. Искомая вероятность равна 3/15. Ответ:0,2                                                                                                     

11. В классе 21 учащийся, среди них два друга — Вадим и Олег. Класс случайным образом разбивают на 3 равные группы. Найдите вероятность того, что Вадим и Олег окажутся в одной группе.                                                                                                        Решение. Пусть один из друзей  находится в некоторой группе. Вместе с ним в группе окажутся 6 человек из 20 оставшихся учащихся. Вероятность того, что друг  окажется среди этих 6 человек, равна 6 : 20 = 0,3.          Ответ: 0,3

12. Перед началом первого тура чемпионата по настольному теннису участников разбивают на игровые пары случайным образом с помощью жребия. Всего в чемпионате участвует 16 спортсменов, среди которых 7 участников из России, в том числе Платон Карпов. Найдите вероятность того, что в первом туре Платон Карпов будет играть с каким-либо спортсменом из России? 6:15=0,4. Ответ:0,4.                                                                       

13. Перед началом первого тура чемпионата по шашкам участников разбивают на игровые пары случайным образом с помощью жребия. Всего в чемпионате участвует 26 шашистов, среди которых 3 участника из России, в том числе Василий Лукин. Найдите вероятность того, что в первом туре Василий Лукин будет играть с каким-либо шашистом из России?   2: 25=0,08. Ответ: 0,08.

14. В классе 26 учащихся, среди них два друга —   Сергей и Андрей. Учащихся случайным образом разбивают на 2 равные группы. Найдите вероятность того, что Сергей и Андрей окажутся в одной группе. Ответ  12 : 25 = 0,48.

15. В классе 21 ученик, среди них 2 друга – Тоша и Гоша. На уроке физкультуры класс случайным образом разбивают на 3 равные группы. Найдите вероятность того, что Тоша и Гоша попали в одну группу. Ответ 6 : 20 = 0,3.

16. В классе 21 учащийся, среди них две подруги — Аня и Нина. Класс случайным образом делят на семь групп, по 3 человека в каждой. Найдите вероятность того, что Аня и Нина окажутся в одной группе.             Ответ: 2: 20 = 0,1.                                                     

17. Механические часы с двенадцатичасовым циферблатом в какой-то момент сломались и перестали идти. Найдите вероятность того, что часовая стрелка остановилась, достигнув отметки 7, но не дойдя до отметки 1.                                                      Ответ. 6 : 12= 0,5 ( 6 делений между 12 и 7, всего 12 делений)

18. Механические часы с двенадцатичасовым циферблатом в какой-то момент сломались и перестали ходить. Найдите вероятность того, что часовая стрелка застыла, достигнув отметки 6, но не дойдя до отметки 9 часов. 3:12 = 0,25

 При решении задач с монетами число всех возможных исходов можно посчитать по формуле  п=2ª, где α –количество бросков

19.   В случайном эксперименте симметричную монету бросают 2 раза. Найдите вероятность того, что орел выпадет ровно 1 раз.

Решение. Всего возможны четыре исхода: решка-решка, решка-орёл, орёл-решка, орёл-орёл. Орёл выпадает ровно один раз в двух случаях, поэтому вероятность того, что орёл выпадет ровно один раз равна  2:4=0,5.  Ответ: 0,5.

20. В случайном эксперименте симметричную монету бросают дважды. Найдите вероятность того, что орёл не выпадет ни разу. Ответ: 1:4=0,25

21. В случайном эксперименте симметричную монету бросают трижды. Найдите вероятность того, что орёл не выпадет ни разу.  Решение. 1:8=0,125  Ответ. 0,125                                                                                    

22. В случайном эксперименте симметричную монету бросают четырежды. Найдите вероятность того, что орёл выпадет ровно 2 раза.                                                                           Решение. Составим список возможных вариантов. Бросают 2 раза может выпасть О — Орел, Р — Решка:
ОО, ОР, РО, РР. Всего 4 исхода из них только один случай удовлетворяет условию.   Вероятность (P) = 1 / 4 = 0.25
.   Ответ: 0.25

23. В случайном эксперименте симметричную монету бросают четырежды. Найдите вероятность того, что решка не выпадет ни разу.                                                                            Решение. Всего исходов   = 16, благоприятных  1 ( ОООО).  1:16 = 0,0625. Ответ: 0,0625                                                                                     

При решении задач с кубиками число всех возможных исходов можно посчитать по формуле  п=6ª, где α –количество бросков 

24. Определите вероятность того, что при бросании игрального кубика (правильной кости) выпадет нечетное число очков.                                                                                                          Решение. При бросании кубика равновозможных  шесть различных исходов. Событию «выпадет нечётное число очков» удовлетворяют три случая: когда на кубике выпадает 1, 3 или 5 очков. Поэтому вероятность того, что на кубике выпадет нечётное число очков равна 3:6=0,5. Ответ: 0,5.

25.  Определите вероятность того, что при бросании кубика выпало число очков, не большее 3.

Решение. При бросании кубика равновозможны шесть различных исходов. Событию «выпадет не больше трёх очков» удовлетворяют три случая: когда на кубике выпадает 1, 2, или 3 очка. Поэтому вероятность того, что на кубике выпадет не больше трёх очков равна  3:6=0,5 Ответ: 0,5.                                                                  

26. Игральную кость бросают дважды. Найдите вероятность того, что оба раза выпало число, большее 3.

Решение. При бросании кубика 6²= 36 различных исходов. Событию «выпадет больше трёх очков» удовлетворяют три случая: когда на кубике выпадает 4, 5, или 6 очков , благоприятных исходов 9 (4,4; 4,5; 4,6; 5,4; 5,5; 5,6;  6,4; 6,5; 6,6.)                       Ответ: 9: 36 = 0,25.

27. В случайном эксперименте бросают три игральные кости. Найдите вероятность того, что в сумме выпадет 7 очков. Результат округлите до сотых.                                      Решение. При бросании кубика 6³= 216  различных исходов, благоприятных  14.  14 : 216 = 0,07.  Ответ: 0,07.     

28.   Коля выбирает трехзначное число. Найдите вероятность того, что оно делится на 5.

Решение. Всего трехзначных чисел 900. На пять делится каждое пятое их них, то есть таких чисел 900:5=180. Вероятность того, что Коля выбрал трехзначное число, делящееся на 5, определяется отношением количества трехзначных чисел, делящихся на 5, ко всему количеству трехзначных чисел: 180:900=0,2.  Ответ: 0,2.

29.Для экзамена подготовили билеты с номерами от 1 до 50. Какова вероятность того, что наугад взятый учеником билет имеет однозначный номер?

Решение. Всего было подготовлено 50 билетов. Среди них 9 были однозначными. Таким образом, вероятность того, что наугад взятый учеником билет имеет однозначный номер равна 9:50=0,18. Ответ: 0,18.

30. В мешке содержатся жетоны с номерами от 5 до 54 включительно. Какова вероятность, того, что извлеченный наугад из мешка жетон содержит двузначное число?

Решение. Всего в мешке жетонов — 50. Среди них 45 имеют двузначный номер. Таким образом, вероятность, того, что извлеченный наугад из мешка жетон содержит двузначное число равна  45 : 50 = 0,9.  Ответ: 0.9. 

31. Какова вероятность того, что случайно выбранное натуральное число от 10 до 19 делится на 3?                                           3 : 10 = 0,3.  Ответ: 0,3.

Противоположные события.

32.  Вероятность того, что новая шариковая ручка пишет плохо (или не пишет), равна 0,19. Покупатель в магазине выбирает одну такую ручку. Найдите вероятность того, что эта ручка пишет хорошо.

Решение. Вероятность того, что ручка пишет хорошо, равна 1 − 0,19 = 0,81.  Ответ: 0,81.

  33. Вероятность того, что в случайный момент времени температура тела здорового человека окажется ниже 36,8°C     равна 0,87. Найдите вероятность того, что в случайный момент времени у здорового человека температура тела окажется 36,8°C или выше.  Ответ. 1-0,87=0,13                                    

34. При изготовлении подшипников диаметром 67 мм вероятность того, что диаметр будет отличаться от заданного не больше, чем на 0,01 мм, равна 0,965. Найдите вероятность того, что случайный подшипник будет иметь диаметр меньше чем 66,99 мм или больше чем 67,01 мм.

Решение. По условию, диаметр подшипника будет лежать в пределах от 66,99 до 67,01 мм с вероятностью 0,965. Поэтому искомая вероятность противоположного события равна 1 − 0,965 = 0,035. Ответ: 0,035.

Несовместные и независимые события.                                                 35. На экзамене по геометрии школьнику достаётся одна задача из сборника. Вероятность того, что эта задача по теме «Углы», равна 0,1. Вероятность того, что это окажется задача по теме «Параллелограмм», равна 0,6. В сборнике нет задач, которые одновременно относятся к этим двум темам. Найдите вероятность того, что на экзамене школьнику достанется задача по одной из этих двух тем.                                          Решение. Суммарная вероятность несовместных событий равна сумме вероятностей этих событий: P=0,6+ 0,1 = 0,7.  Ответ: 0,7.

36. Вероятность того, что на тесте по биологии учащийся О. верно решит больше 11 задач, равна 0,67. Вероятность того, что О. верно решит больше 10 задач, равна 0,74. Найдите вероятность того, что О. верно решит ровно 11 задач.

Решение. Рассмотрим события A = «учащийся решит 11 задач» и В = «учащийся решит больше 11 задач». Их сумма — событие A + B = «учащийся решит больше 10 задач». События A и В несовместные, вероятность их суммы равна сумме вероятностей этих событий:  P(A + B) = P(A) + P(B). Тогда, используя данные задачи, получаем: 0,74 = P(A) + 0,67, откуда P(A) = 0,74 − 0,67 = 0,07. Ответ: 0,07.

                                                                                                                       37. Вероятность того, что на тесте по химии учащийся П. верно решит больше 8 задач, равна 0,48. Вероятность того, что П. верно решит больше 7 задач, равна 0,54. Найдите вероятность того, что П. верно решит ровно 8 задач. Решение. Вероятность решить несколько задач складывается из суммы вероятностей решить каждую из этих задач. Больше 8:  решить 9-ю, 10-ю … Больше 7:  решить 8-ю, 9-ю, 10-ю …Вероятность решить 8-ю = 0,54-0,48=0,06.  Ответ:0.06

38. На клавиатуре телефона 10 цифр, от 0 до 9. Какова вероятность того, что случайно нажатая цифра будет меньше 4?    Ответ: 4 : 10 = 0,4.

 39. Биатлонист пять раз стреляет по мишеням. Вероятность попадания в мишень при одном выстреле равна 0,8. Найдите вероятность того, что биатлонист первые три раза попал в мишени, а последние два промахнулся. Результат округлите до сотых.

Решение. Поскольку биатлонист попадает в мишени с вероятностью 0,8, он промахивается с вероятностью 1 − 0,8 = 0,2. Cобытия попасть или промахнуться при каждом выстреле независимы, вероятность произведения независимых событий равна произведению их вероятностей. Тем самым, вероятность события «попал, попал, попал, промахнулся, промахнулся» равна 0,8•0,8•0,8•0,2•0,2=0,02048.  Ответ:0.02048.

40. Помещение освещается фонарём с двумя лампами. Вероятность перегорания лампы в течение года равна 0,3. Найдите вероятность того, что в течение года хотя бы одна лампа не перегорит.

Решение. Найдем вероятность того, что перегорят обе лампы. Эти события независимые, вероятность их произведения равно произведению вероятностей этих событий: 0,3·0,3 = 0,09. Событие, состоящее в том, что не перегорит хотя бы одна лампа, противоположное. Следовательно, его вероятность равна 1 − 0,09 = 0,91.   Ответ: 0,91.

41. Вероятность того, что батарейка бракованная, равна 0,06. Покупатель в магазине выбирает случайную упаковку, в которой две таких батарейки. Найдите вероятность того, что обе батарейки окажутся исправными.

Решение. Вероятность того, что батарейка исправна, равна 0,94. Вероятность произведения независимых событий (обе батарейки окажутся исправными) равна произведению вероятностей этих событий: 0,94·0,94 = 0,8836.   Ответ: 0,8836.

42.   Если гроссмейстер А. играет белыми, то он выигрывает у гроссмейстера Б. с вероятностью 0,52. Если А. играет черными, то А. выигрывает у Б. с вероятностью 0,3. Гроссмейстеры А. и Б. играют две партии, причем во второй партии меняют цвет фигур. Найдите вероятность того, что А. выиграет оба раза.

Решение. Возможность выиграть первую и вторую партию не зависят друг от друга. Вероятность произведения независимых событий равна произведению их вероятностей: 0,52 · 0,3 = 0,156.   Ответ: 0,156.

43.   В магазине три продавца. Каждый из них занят с клиентом с вероятностью 0,3. Найдите вероятность того, что в случайный момент времени все три продавца заняты одновременно (считайте, что клиенты заходят независимо друг от друга).

Решение. Вероятность произведения независимых событий равна произведению вероятностей этих событий. Поэтому вероятность того, что все три продавца заняты равна                       (0,3)³ = 0,027.    Ответ: 0,027.

   44. Из районного центра в деревню ежедневно ходит автобус. Вероятность того, что в понедельник в автобусе окажется меньше 20 пассажиров, равна 0,94. Вероятность того, что окажется меньше 15 пассажиров, равна 0,56. Найдите вероятность того, что число пассажиров будет от 15 до 19.

Решение. Рассмотрим события A = «в автобусе меньше 15 пассажиров» и  В = «в автобусе от 15 до 19 пассажиров». Их сумма — событие A + B = «в автобусе меньше 20 пассажиров». События A и  В несовместные, вероятность их суммы равна сумме вероятностей этих событий: P(A + B) = P(A) + P(B).

 Тогда, используя данные задачи, получаем: 0,94 = 0,56 + P(В), откуда P(В) = 0,94 − 0,56 = 0,38.Ответ: 0,38.

45. На экзамене по геометрии школьнику достаётся один вопрос из списка экзаменационных вопросов. Вероятность того, что это вопрос на тему «Вписанная окружность», равна 0,2. Вероятность того, что это вопрос на тему «Параллелограмм», равна 0,15. Вопросов, которые одновременно относятся к этим двум темам, нет. Найдите вероятность того, что на экзамене школьнику достанется вопрос по одной из этих двух тем.

Решение. Вероятность суммы двух несовместных событий равна сумме вероятностей этих событий: 0,2 + 0,15 = 0,35.

Ответ: 0,35.

 46.Вероятность того, что новый электрический чайник прослужит больше года, равна 0,97. Вероятность того, что он прослужит больше двух лет, равна 0,89. Найдите вероятность того, что он прослужит меньше двух лет, но больше года.

Решение. Пусть A = «чайник прослужит больше года, но меньше двух лет», В = «чайник прослужит больше двух лет», С = «чайник прослужит ровно два года», тогда A + B + С = «чайник прослужит больше года». События A, В и С несовместные, вероятность их суммы равна сумме вероятностей этих событий. Вероятность события С, состоящего в том, что чайник выйдет из строя ровно через два года — строго в тот же день, час и секунду — равна нулю. Тогда:  P(A + B+ С) = P(A) + P(B)+ P(С)= P(A) + P(B)

 откуда, используя данные из условия, получаем     0,97 = P(A) + 0,89.Тем самым, для искомой вероятности имеем: P(A) = 0,97 − 0,89 = 0,08.   Ответ: 0,08.

 

 47. В Волшебной стране бывает два типа погоды: хорошая и отличная, причём погода, установившись утром, держится неизменной весь день. Известно, что с вероятностью 0,8 погода завтра будет такой же, как и сегодня. Сегодня 3 июля, погода в Волшебной стране хорошая. Найдите вероятность того, что 6 июля в Волшебной стране будет отличная погода.

Решение. Для погоды на 4, 5 и 6 июля есть 4 варианта: ХХО, ХОО, ОХО, ООО (здесь Х — хорошая, О — отличная погода). Найдем вероятности наступления такой погоды: P(XXO) = 0,8·0,8·0,2 = 0,128; P(XOO) = 0,8·0,2·0,8 = 0,128; P(OXO) = 0,2·0,2·0,2 = 0,008;  P(OOO) = 0,2·0,8·0,8 = 0,128.Указанные события несовместные, вероятность их суммы равна сумме вероятностей этих событий:

 P(ХХО) + P(ХОО) + P(ОХО) + P(ООО) = 0,128 + 0,128 + 0,008 + 0,128 = 0,392.   Ответ: 0,392.

48. В магазине стоят два платёжных автомата. Каждый из них может быть неисправен с вероятностью 0,05 независимо от другого автомата. Найдите вероятность того, что хотя бы один автомат исправен. 

Решение. Найдем вероятность того, что неисправны оба автомата. Эти события независимые, вероятность их произведения равна произведению вероятностей этих событий: 0,05 · 0,05 = 0,0025. Событие, состоящее в том, что исправен хотя бы один автомат, противоположное. Следовательно, его вероятность равна 1 − 0,0025 = 0,9975. Ответ: 0,9975.

 49. В торговом центре два одинаковых автомата продают кофе. Вероятность того, что к концу дня в автомате закончится кофе, равна 0,3. Вероятность того, что кофе закончится в обоих автоматах, равна 0,12. Найдите вероятность того, что к концу дня кофе останется в обоих автоматах.

Решение. Рассмотрим событиеА = кофе закончится в первом автомате, В = кофе закончится во втором автомате.

Вероятность того, что кофе останется в первом автомате равна 1 − 0,3 = 0,7. Вероятность того, что кофе останется во втором автомате равна 1 − 0,3 = 0,7. Вероятность того, что кофе останется в первом или втором автомате равна 1 − 0,12 = 0,88. Поскольку P(A + B) = P(A) + P(B) − P(A·B), имеем: 0,88 = 0,7 + 0,7 − х, откуда искомая вероятность х = 0,52. Ответ: 0,9975.

50. Две фабрики выпускают одинаковые стекла для автомобильных фар. Первая фабрика выпускает 45% этих стекол, вторая — 55%. Первая фабрика выпускает 3% бракованных стекол, а вторая — 1%. Найдите вероятность того, что случайно купленное в магазине стекло окажется бракованным.

Решение. Вероятность того, что стекло куплено на первой фабрике и оно бракованное: 0,45 · 0,03 = 0,0135. Вероятность того, что стекло куплено на второй фабрике и оно бракованное: 0,55 · 0,01 = 0,0055.  Поэтому по формуле полной вероятности вероятность того, что случайно купленное в магазине стекло окажется бракованным равна 0,0135 + 0,0055 = 0,019.                       Ответ: 0,019.

51. Ковбой Джон попадает в муху на стене с вероятностью 0,9, если стреляет из пристрелянного револьвера. Если Джон стреляет из непристрелянного револьвера, то он попадает в муху с вероятностью 0,2. На столе лежит 10 револьверов, из них только 4 пристрелянные. Ковбой Джон видит на стене муху, наудачу хватает первый попавшийся револьвер и стреляет в муху. Найдите вероятность того, что Джон промахнётся.

Решение. Джон попадает в муху, если схватит пристрелянный револьвер и попадет из него, или если схватит непристрелянный револьвер и попадает из него. По формуле условной вероятности, вероятности этих событий равны соответственно 0,4·0,9 = 0,36 и 0,6·0,2 = 0,12. Эти события несовместны, вероятность их суммы равна сумме вероятностей этих событий: 0,36 + 0,12 = 0,48. Событие, состоящее в том, что Джон промахнется, противоположное. Его вероятность равна 1 − 0,48 = 0,52. Ответ. 0,52

 

 52. Чтобы поступить в институт на специальность «Лингвистика», абитуриент должен набрать на ЕГЭ не менее 70 баллов по каждому из трёх предметов — математика, русский язык и иностранный язык. Чтобы поступить на специальность «Коммерция», нужно набрать не менее 70 баллов по каждому из трёх предметов — математика, русский язык и обществознание.

Вероятность того, что абитуриент З. получит не менее 70 баллов по математике, равна 0,6, по русскому языку — 0,8, по иностранному языку — 0,7 и по обществознанию — 0,5.

Найдите вероятность того, что З. сможет поступить хотя бы на одну из двух упомянутых специальностей.

Решение. В силу независимости событий, вероятность успешно сдать экзамены на лингвистику: 0,6·0,8·0,7 = 0,336, вероятность успешно сдать экзамены на коммерцию: 0,6·0,8·0,5 = 0,24, вероятность успешно сдать экзамены и на «Лингвистику», и на «Коммерцию»: 0,6·0,8·0,7·0,5 = 0,168. Успешная сдача экзаменов на «Лингвистику» и на «Коммерцию» — события совместные, поэтому вероятность их суммы равна сумме вероятностей этих событий, уменьшенной на вероятность их произведения. Тем самым, поступить на одну из этих специальностей абитуриент может с вероятностью 0,336 + 0,24 − 0,168 = 0,408.  Ответ: 0,408.

53. По отзывам покупателей Иван Иванович оценил надёжность двух интернет- магазинов. Вероятность того, что нужный товар доставят из магазина А, равна 0,8. Вероятность того, что этот товар доставят из магазина Б, равна 0,9. Иван Иванович заказал товар сразу в обоих магазинах. Считая, что интернет-магазины работают независимо друг от друга, найдите вероятность того, что ни один магазин не доставит товар.                                                                                          Решение. Вероятность того, что первый магазин не доставит товар равна 1 − 0,9 = 0,1. Вероятность того, что второй магазин не доставит товар равна 1 − 0,8 = 0,2. Поскольку эти события независимы, вероятность их произведения (оба магазина не доставят товар) равна произведению вероятностей этих событий: 0,1 · 0,2 = 0,02.  Ответ: 0,02.

54.Перед началом волейбольного матча капитаны команд тянут честный жребий, чтобы определить, какая из команд начнёт игру с мячом. Команда «Статор» по очереди играет с командами «Ротор», «Мотор» и «Стартер». Найдите вероятность того, что «Статор» будет начинать только первую и последнюю игры. Решение. Требуется найти вероятность произведения трех событий: «Статор» начинает первую игру, не начинает вторую игру, начинает третью игру. Вероятность произведения независимых событий равна произведению вероятностей этих событий. Вероятность каждого из них равна 0,5, откуда находим: 0,5·0,5·0,5 = 0,125.   Ответ: 0,125.

  

55. Всем пациентам с подозрением на гепатит делают анализ крови. Если анализ выявляет гепатит, то результат анализа называется положительным. У больных гепатитом пациентов анализ даёт положительный результат с вероятностью 0,9. Если пациент не болен гепатитом, то анализ может дать ложный положительный результат с вероятностью 0,01. Известно, что 5% пациентов, поступающих с подозрением на гепатит, действительно больны гепатитом. Найдите вероятность того, что результат анализа у пациента, поступившего в клинику с подозрением на гепатит, будет положительным.

Решение.  Анализ пациента может быть положительным по двум причинам: А) пациент болеет гепатитом, его анализ верен; B) пациент не болеет гепатитом, его анализ ложен. Это несовместные события, вероятность их суммы равна сумме вероятностей этих событий. Имеем: Р(А)=0,9•0.05=0,045;                                         Р(В)= 0,01•0,95=0,0095  ,Р(А+В)=Р(А)(В)=0,045+0,0095=0,0545.

 Ответ:0,0545.

  56. Автоматическая линия изготавливает батарейки. Вероятность того, что готовая батарейка неисправна, равна 0,02. Перед упаковкой каждая батарейка проходит систему контроля. Вероятность того, что система забракует неисправную батарейку, равна 0,99. Вероятность того, что система по ошибке забракует исправную батарейку, равна 0,01. Найдите вероятность того, что случайно выбранная батарейка будет забракована системой контроля.

Решение. Ситуация, при которой батарейка будет забракована, может сложиться в результате событий: A = батарейка действительно неисправна и забракована справедливо или В = батарейка исправна, но по ошибке забракована. Это несовместные события, вероятность их суммы равна сумме вероятностей эти событий. Имеем: Р(А+В)=Р(А)+Р(В)=0,02•0,99+0,98•0,01=0,0198+0,0098=0,0296 Ответ: 0,0296.

 57. Стрелок стреляет по мишени один раз. В случае промаха стрелок делает второй выстрел по той же мишени. Вероятность попасть в мишень при одном выстреле равна 0,7. Найдите вероятность того, что мишень будет поражена (либо первым, либо вторым выстрелом).

Решение. Пусть A — событие, состоящее в том, что мишень поражена стрелком с первого выстрела, B — событие, состоящее в том, что мишень поражена со второго выстрела. Вероятность события A равна P(A) = 0,7. Событие B наступает, если, стреляя первый раз, стрелок промахнулся, а, стреляя второй раз, попал. Это независимые события, их вероятность равна произведению вероятностей этих событий: P(B) = 0,3·0,7 = 0,21. События A и B несовместные, вероятность их суммы равна сумме вероятностей этих событий:  P (A + B) = P(A) + P(B) = 0,7 + 0,21 = 0,91.   Ответ: 0,91.

58.Перед началом футбольного матча судья бросает монетку, чтобы определить, какая из команд будет первой владеть мячом. Команда А должна сыграть два матча — с командой В и с командой С. Найдите вероятность того, что в обоих матчах первой мячом будет владеть команда А.

Решение. Рассмотрим все возможные исходы жеребьёвки.

 · КомандаА в матче в обоих матчах первой владеет мячом.

 · КомандаА в матче в обоих матчах не владеет мячом первой.

 · КомандаА в матче с командой В владеет мячом первой, а в матче с командой С — второй.

 · КомандаА в матче с командой С владеет мячом первой, а в матче с командой В — второй.

Из четырех исходов один является благоприятным, вероятность его наступления равна 1:4=0,25. Ответ: 0,25.

59. Стрелок 4 раза стреляет по мишеням. Вероятность попадания в мишень при одном выстреле равна 0,5. Найдите вероятность того, что стрелок первые 3 раза попал в мишени, а последний раз промахнулся.

Решение. Вероятность промаха равна 1 − 0,5 = 0,5. Вероятность того, что стрелок первые три раза попал в мишени равна 0,53 = 0,125. Откуда, вероятность события, при котором стрелок сначала три раза попадает в мишени, а четвёртый раз промахивается равна 0,125 · 0,5 = 0,0625.     Ответ: 0,0625.

60. Перед началом матча по футболу судья бросает монету, чтобы определить, какая из команд будет первой владеть мячом. Команда «Байкал» играет по очереди с командами

«Амур», «Енисей», «Иртыш». Найти вероятность того, что команда «Байкал» будет первой  владеть мячом только в игре с «Амуром».

Решение. Монету бросают 3 раза.

Для команды «Байкал» возможные исходы в трех бросках {О О О},{Р О О}, {О Р О}, {О О Р},                  {Р Р О},{Р О Р}, {О Р Р},{Р Р Р}. Всего исходов 8, благоприятныx1(выпадение орла в первой игре)  {О Р Р, 1:8=0,125.Ответ 0,125.

61.У Пети в кармане лежат  шесть монет: четыре монеты по рублю и две монеты по два рубля. Петя, не глядя, переложил какие-то три монеты в другой карман. Найдите вероятность  того, что теперь две двухрублевые монеты лежат в одном кармане.

 Решение. Пронумеруем монеты: рублевые – 1, 2, 3, 4;  двухрублевые – 5, 6. {123} {124} {125} {126} {134} {135} {136} {145} {146} {156} {234} {235} {236} {245} {246} {256} {345} {346} {356} {456} 

 n = 20     – число всех исходов .Взять три монеты можно так: (числа в порядке возрастания,чтобы не пропустить комбинацию)  m = 8  – число благоприятных  исходов

(комбинации, в которых монеты 5 и 6 (двухрублевые) не взяты или взяты обе. 8:20=0,4

4. Введение в теорию вероятностей


1. Вспоминай формулы по каждой теме


2. Решай новые задачи каждый день


3. Вдумчиво разбирай решения

Сложные задачи по теории вероятности

Общая памятка по всем разделам теории вероятностей:


Задание
1

#3858

Уровень задания: Равен ЕГЭ

Чтобы пройти в следующий круг соревнований, футбольной команде нужно набрать хотя бы (4) очка в двух играх. Если команда выигрывает, она получает (3) очка, в случае ничьей — (1) очко, если проигрывает — (0) очков. Найдите вероятность того, что команде удастся выйти в следующий круг соревнований. Считайте, что в каждой игре вероятности выигрыша и проигрыша одинаковы и равны (0,3).

Чтобы команда в двух играх набрала не менее (4) очков, ей нужно: либо 1) выиграть обе игры, либо 2) выиграть в одной из игр и сыграть вничью в другой игре.
Так как вероятности выиграть и проиграть одинакова и равна (0,3), то вероятность сыграть вничью равна (1-0,3-0,3=0,4).
Следовательно, вероятности в этих случаях равны соответственно:
1) (0,3cdot 0,3)
2) (0,3cdot 0,4+0,4cdot 0,3) (выиграть в первой игре и сыграть вничью во второй или сыграть вничью в первой и выиграть во второй).
Следовательно, вероятность того, что команда выйдет в следующий круг соревнований, равна [0,3cdot 0,3+0,3cdot 0,4+0,4cdot 0,3=0,33]

Ответ: 0,33


Задание
2

#2739

Уровень задания: Сложнее ЕГЭ

Илья решает задачу по геометрии, в которой дан четырёхугольник (ABCD), причём (AB = 5), (BC = 6), (CD = 4), (AD = 10). В условии задачи сказано, что одна из вершин является центром некоторой окружности и Илья думает, какую вершину ему выбрать в качестве центра этой самой окружности.

Известно, что вероятность выбора каждой конкретной вершины пропорциональна сумме длин сторон четырёхугольника (ABCD), проходящих через эту вершину. Какова вероятность того, что Илья выберет вершину (B)?

Через вершину (A) проходят стороны (AB) и (AD), их сумма: (AB + AD = 15).

Через вершину (B) проходят стороны (AB) и (BC), их сумма: (AB + BC = 11).

Через вершину (C) проходят стороны (BC) и (CD), их сумма: (BC + CD = 10).

Через вершину (D) проходят стороны (CD) и (DA), их сумма: (CD + DA = 14).

Обозначим вероятность выбора вершины (A) через (P(A)) (для остальных вершин аналогично). Тогда по условию имеем: [P(A) = 15k,qquad P(B) = 11k,qquad P(C) = 10k,qquad P(D) = 14k,,] но (P(A) + P(B) + P(C) + P(D) = 1), тогда (k = 0,02), откуда находим: (P(B) = 0,22).

Ответ: 0,22


Задание
3

#191

Уровень задания: Сложнее ЕГЭ

Монетку подбросили 10 раз. Какова вероятность того, что выпало не менее 9 орлов? Ответ округлите до тысячных.

Условие того, что выпало не менее 9 орлов эквивалентно тому, что выпало не более 1 решки, то есть либо ровно 1 решка, либо 0 решек.

Количество всевозможных различных исходов в серии из 10 испытаний равно (2^{10} = 1024).

Среди них есть 11 исходов, подходящих под условие: (Орёл; Орёл; …; Орёл), (Орёл; Орёл; …; Орёл; Решка), (Орёл; Орёл; …; Решка; Орёл), …, (Решка; Орёл; …; Орёл), следовательно, искомая вероятность равна [dfrac{11}{1024}.] После округления получим (0,011).

Ответ: 0,011


Задание
4

#190

Уровень задания: Сложнее ЕГЭ

Монетку подбросили 3 раза. Какова вероятность того, что выпало не менее 3 орлов? Ответ округлите до тысячных.

Условие того, что выпало не менее 3 орлов эквивалентно тому, что выпали только орлы.

Количество всевозможных различных исходов в серии из 3 испытаний равно (2^3 = 8) . Среди них есть ровно один исход, подходящий под условие: (Орёл; Орёл; Орёл). Таким образом, искомая вероятность равна [dfrac{1}{8} = 0,125.]

Ответ: 0,125


Задание
5

#189

Уровень задания: Сложнее ЕГЭ

Монетку подбросили 2 раза. Какова вероятность того, что выпало не менее 1 орла? Ответ округлите до тысячных.

Всевозможных исходов в серии из 2 подбрасываний может быть (2^2 = 4): (Орёл; Орёл), (Орёл; Решка), (Решка; Орёл), (Решка; Решка).

Среди выписанных (всевозможных) исходов под условие задачи подходят первые 3, следовательно, искомая вероятность равна [dfrac{3}{4} = 0,75.]

Ответ: 0,75


Задание
6

#2658

Уровень задания: Сложнее ЕГЭ

Игорь трижды подбрасывает правильную игральную кость. Какова вероятность того, что за эти три подбрасывания ровно один раз выпадет число, кратное трём, а сумма результатов подбрасываний не будет делиться на (3)? Ответ округлите до сотых.

Так как игральная кость правильная, то вероятность выпадения каждой грани равна (dfrac{1}{6}). Среди чисел на гранях есть два числа, дающих при делении на (3) остаток (0), два числа, дающих при делении на (3) остаток (1) и два числа, дающих при делении на (3) остаток (2).

Тогда вероятность за одно подбрасывание получить, например, число, дающее при делении на (3) остаток (1), равна (dfrac{1}{3}). С другими остатками аналогично.

Условие задачи можно переформулировать в следующем виде: какова вероятность за три подбрасывания получить результаты, остатки от деления на (3) которых будут содержать единственный (0) и два одинаковых числа?

Таким образом, нас устраивают исходы, остатки от деления на (3) которых будут иметь вид:

[begin{aligned}
&0,quad 1,quad 1\
&1,quad 0,quad 1\
&1,quad 1,quad 0\
&0,quad 2,quad 2\
&2,quad 0,quad 2\
&2,quad 2,quad 0,.
end{aligned}]

Вероятность любого из выписанных исходов равна [dfrac{1}{3}cdot dfrac{1}{3}cdot dfrac{1}{3},.] При этом различных исходов здесь шесть, следовательно, вероятность получения подходящего исхода равна [6cdot dfrac{1}{3}cdot dfrac{1}{3}cdot dfrac{1}{3} = dfrac{2}{9},.] После округления получим ответ (0,22).

Ответ: 0,22


Задание
7

#2765

Уровень задания: Сложнее ЕГЭ

Таня заметила, что в казино “Подкинем” используют неправильную игральную кость (т.е. не у всех граней вероятности выпадения одинаковы). При этом она установила, что вероятность выпадения чётного числа равна (0,6); вероятность выпадения числа, делящегося на (3), равна (0,3); вероятность того, что выпадет (1) или (5), равна (0,22). Найдите вероятность того, что на этой игральной кости выпадет число (3). Ответ округлите до сотых.

Вероятность выпадения числа (n) обозначим через (P({n})), вероятность выпадения одного из чисел (m) и (n) обозначим через (P({m; n})), а вероятность выпадения одного из чисел (m), (n) и (k) обозначим через (P({m; n; k})). Тогда [P({2; 4; 6}) = 0,6qquadLeftrightarrowqquad P({1; 3; 5}) = 1 — 0,6 = 0,4]

При этом (P({1; 5}) = 0,22), но ведь (P({1; 3; 5}) — P({1; 5}) = P({3})), следовательно, [P({3}) = 0,4 — 0,22 = 0,18,.]

Ответ: 0,18

Если выпускник готовится к сдаче ЕГЭ по математике профильного уровня, ему необходимо научиться решать задачи на применение теории вероятности повышенной сложности. Как показывает практика многих лет, такие задания являются обязательной частью программы аттестационного испытания. Поэтому если учащийся не до конца понимает принцип решения сложных задач на теорию вероятности, ему обязательно стоит вновь разобраться в данной теме.

Вместе с образовательным порталом «Школково» старшеклассники смогут качественно подготовиться к прохождению аттестационного испытания. Наш сайт позволит определить наиболее сложные темы и восполнить пробелы в знаниях. Опытные специалисты «Школково» подготовили весь необходимый материал, изложив его таким образом, чтобы школьники с любым уровнем подготовки смогли легко справиться с решением сложных задач ЕГЭ на теорию вероятности. Базовая информация по данной теме представлена в разделе «Теоретическая справка».

Чтобы попрактиковаться в выполнении сложных задач ЕГЭ по теории вероятности, школьники могут выполнить соответствующие упражнения. Простые и сложные задания, подобранные нашими специалистами, содержат подробные алгоритмы решения и правильные ответы. База заданий регулярно обновляется и дополняется.

Выполнять упражнения школьники из Москвы и других российских городов могут в онлайн-режиме. При необходимости задания по теории вероятности в ЕГЭ можно сохранить в разделе «Избранное». Благодаря этому вы сможете быстро найти интересующие примеры и обсудить алгоритмы нахождения правильного ответа с преподавателем.

Курс Глицин. Любовь, друзья, спорт и подготовка к ЕГЭ

Курс Глицин. Любовь, друзья, спорт и подготовка к ЕГЭ

Задание 3. Теория вероятностей на ЕГЭ по математике.

Мы начнем с простых задач и основных понятий теории вероятностей.
Случайным называется событие, которое нельзя точно предсказать заранее. Оно может либо произойти, либо нет.
Вы выиграли в лотерею — случайное событие. Пригласили друзей отпраздновать выигрыш, а они по дороге к вам застряли в лифте — тоже случайное событие. Правда, мастер оказался поблизости и освободил всю компанию через десять минут — и это тоже можно считать счастливой случайностью…

Наша жизнь полна случайных событий. О каждом из них можно сказать, что оно произойдет с некоторой вероятностью. Скорее всего, вы интуитивно знакомы с этим понятием. Теперь мы дадим математическое определение вероятности.

Начнем с самого простого примера. Вы бросаете монетку. Орел или решка?

Такое действие, которое может привести к одному из нескольких результатов, в теории вероятностей называют испытанием.

Орел и решка — два возможных исхода испытания.

Орел выпадет в одном случае из двух возможных. Говорят, что вероятность того, что монетка упадет орлом, равна 1/2.

Бросим игральную кость. У кубика шесть граней, поэтому возможных исходов тоже шесть.

Например, вы загадали, что выпадет три очка. Это один исход из шести возможных. В теории вероятностей он будет называться благоприятным исходом.

Вероятность выпадения тройки равна 1/6 (один благоприятный исход из шести возможных).

Вероятность четверки — тоже 1/6.

А вот вероятность появления семерки равна нулю. Ведь грани с семью точками на кубике нет.

Вероятность события равна отношению числа благоприятных исходов к общему числу исходов.

Очевидно, что вероятность не может быть больше единицы.

Вот другой пример. В пакете 25 яблок, из них 8 — красные, остальные — зеленые. Ни формой, ни размером яблоки не отличаются. Вы запускаете в пакет руку и наугад вынимаете яблоко. Вероятность вытащить красное яблоко равна 8/25, а зеленое — 17/25.

Вероятность достать красное или зеленое яблоко равна 8/25+17/25=1.
 

БЕСПЛАТНЫЙ МИНИ-КУРС ПО ТЕОРВЕРУ

Определение вероятности. Простые задачи из вариантов ЕГЭ.

Разберем задачи по теории вероятностей, входящие в сборники для подготовки к ЕГЭ.

1. В фирме такси в данный момент свободно 15 машин: 2 красных, 9 желтых и 4 зеленых. По вызову выехала одна из машин, случайно оказавшихся ближе всего к заказчице. Найдите вероятность того, что к ней приедет желтое такси.

Всего имеется 15 машин, то есть к заказчице приедет одна из пятнадцати. Желтых — девять, и значит, вероятность приезда именно желтой машины равна 9/15, то есть 0,6.

2. В сборнике билетов по биологии всего 25 билетов, в двух из них встречается вопрос о грибах. На экзамене школьнику достаётся один случайно выбранный билет. Найдите вероятность того, что в этом билете не будет вопроса о грибах.

Очевидно, вероятность вытащить билет без вопроса о грибах равна 23/25, то есть 0,92.

3. Родительский комитет закупил 30 пазлов для подарков детям на окончание учебного года, из них 12 с картинами известных художников и 18 с изображениями животных. Подарки распределяются случайным образом. Найдите вероятность того, что Вовочке достанется пазл с животным.

Задача решается аналогично.

Ответ: 0,6.

4. В чемпионате по гимнастике участвуют 20 спортсменок: 8 — из России, 7 — из США, остальные — из Китая. Порядок, в котором выступают гимнастки, определяется жребием. Найдите вероятность того, что спортсменка, выступающая последней, окажется из Китая.

Давайте представим, что все спортсменки одновременно подошли к шляпе и вытянули из нее бумажки с номерами. Кому-то из них достанется двадцатый номер. Вероятность того, что его вытянет китайская спортсменка, равен 5/20 (поскольку из Китая — 5 спортсменок). Ответ: 0,25.

5. Ученика попросили назвать число от 1 до 100. Какова вероятность того, что он назовет число кратное пяти?

1,2,3,4,5,6,7,8,9,10,11 dotsc 100.

Каждое пятое число из данного множества делится на 5. Значит, вероятность равна 1/5.

6. Брошена игральная кость. Найдите вероятность того, что выпадет нечетное число очков.

1, 3, 5 — нечетные числа; 2,4,6 — четные. Вероятность нечетного числа очков равна 1/2.

Ответ: 0,5.

7. Монета брошена три раза. Какова вероятность двух «орлов» и одной «решки»?

Заметим, что задачу можно сформулировать по-другому: бросили три монеты одновременно. На решение это не повлияет.

Как вы думаете, сколько здесь возможных исходов?

Бросаем монету. У этого действия два возможных исхода: орел и решка.

Две монеты — уже четыре исхода:

орел орел
орел решка
решка орел
решка решка

Три монеты? Правильно, 8. исходов, так как 2 cdot 2 cdot 2 = 2^3=8.

Вот они:

орел орел орел
орел орел решка
орел решка орел
решка орел орел
орел решка решка
решка орел решка
решка решка орел
решка решка решка

Два орла и одна решка выпадают в трех случаях из восьми.

Ответ: 3/8.

8. В случайном эксперименте бросают две игральные кости. Найдите вероятность того, что в сумме выпадет 8 очков. Результат округлите до сотых.

Бросаем первую кость — шесть исходов. И для каждого из них возможны еще шесть — когда мы бросаем вторую кость.

Получаем, что у данного действия — бросания двух игральных костей — всего 36 возможных исходов, так как 6^2=36.

А теперь — благоприятные исходы:

2 6

3 5

4 4

5 3

6 2

Вероятность выпадения восьми очков равна 5/36 approx 0,14.

9. Стрелок попадает в цель с вероятностью 0,9. Найдите вероятность того, что он попадёт в цель четыре выстрела подряд.

Если вероятность попадания равна 0,9 — следовательно, вероятность промаха 0,1. Рассуждаем так же, как и в предыдущей задаче. Вероятность двух попадания подряд равна 0,9 cdot 0,9=0,81. А вероятность четырех попаданий подряд равна 0,9 cdot 0,9 cdot 0,9 cdot 0,9 = 0,6561.

Лень разбираться самому?
Присоединяйся к мини-курсу по теории вероятностей

ПОДРОБНЕЕ

Вероятность: логика перебора.

10. В кармане у Пети было 2 монеты по 5 рублей и 4 монеты по 10 рублей. Петя не глядя переложил какие-то 3 монеты в другой карман. Найдите вероятность того, что пятирублевые монеты лежат теперь в разных карманах.

Мы знаем, что вероятность события равна отношению числа благоприятных исходов к общему числу исходов. Но как посчитать все эти исходы?

Можно, конечно, обозначить пятирублевые монеты цифрами 1, а десятирублевые цифрами 2 — а затем посчитать, сколькими способами можно выбрать три элемента из набора 1 1 2 2 2 2.

Однако есть более простое решение:

Кодируем монеты числами: 1, 2 (это пятирублёвые), 3, 4, 5, 6 (это десятирублёвые). Условие задачи можно теперь сформулировать так:

Есть шесть фишек с номерами от 1 до 6. Сколькими способами можно разложить их по двум карманам поровну, так чтобы фишки с номерами 1 и 2 не оказались вместе?

Давайте запишем, что у нас в первом кармане.

Для этого составим все возможные комбинации из набора 1 2 3 4 5 6. Набор из трёх фишек будет трёхзначным числом. Очевидно, что в наших условиях 1 2 3 и 2 3 1 — это один и тот же набор фишек. Чтобы ничего не пропустить и не повториться, располагаем соответствующие трехзначные числа по возрастанию:

123, 124, 125, 126

А дальше? Мы же говорили, что располагаем числа по возрастанию. Значит, следующее — 134, а затем:

135, 136, 145, 146, 156.

Все! Мы перебрали все возможные комбинации, начинающиеся на 1. Продолжаем:

234, 235, 236, 245, 246, 256, 345, 346, 356, 456.

Всего 20 возможных исходов.

У нас есть условие — фишки с номерами 1 и 2 не должны оказаться вместе. Это значит, например, что комбинация 356 нам не подходит — она означает, что фишки 1 и 2 обе оказались не в первом, а во втором кармане. Благоприятные для нас исходы — такие, где есть либо только 1, либо только 2. Вот они:

134, 135, 136, 145, 146, 156, 234, 235, 236, 245, 246, 256 – всего 12 благоприятных исходов.

Тогда искомая вероятность равна 12/20.

Ответ: 0,6.

Сумма событий, произведение событий и их комбинации

11. Вероятность того, что новый электрический чайник прослужит больше года, равна 0,93. Вероятность того, что он прослужит больше двух лет, равна 0,87. Найдите вероятность того, что он прослужит меньше двух лет, но больше года.

Проработав год, чайник может либо сломаться на второй год, либо благополучно служить и после 2 лет работы.
Пусть p – вероятность того, что чайник прослужил больше года.

p_1 – вероятность того, что он сломается на второй год, p_2 – вероятность того, что он прослужит больше двух лет.

Очевидно, p= p_1+p_2.

Тогда p_1=p-p_2=0,93-0,87=0,06.

Ответ: 0,06.

События, взаимоисключающие друг друга в рамках данной задачи, называются несовместными. Появление одного из несовместных событий исключает появление других.

Сумма двух событий – термин, означающий, что произошло или первое событие, или второе, или оба сразу.

Вероятность суммы несовместных событий равна сумме их вероятностей.

В нашей задаче события «чайник сломался на второй год работы» и «чайник работает больше двух лет» — несовместные. Чайник или сломался, или остается в рабочем состоянии.

12. На рисунке изображён лабиринт. Паук заползает в лабиринт в точке «Вход». Развернуться и ползти назад паук не может. На каждом разветвлении паук выбирает путь, по которому ещё не полз. Считая выбор дальнейшего пути случайным, определите, с какой вероятностью паук выйдет через выход А.

Пронумеруем развилки, на которых паук может случайным образом свернуть в ту или другую сторону.

Он может либо выйти в выход D, и вероятность этого события равна frac{1}{2}. Либо уйти дальше в лабиринт. На второй развилке он может либо свернуть в тупик, либо выйти в выход В (с вероятностью frac{1}{2}cdot frac{1}{2}=frac{1}{4}). На каждой развилке вероятность свернуть в ту или другую сторону равна frac{1}{2}, а поскольку развилок пять, вероятность выбраться через выход А равна frac{1}{32}, то есть 0,03125.

События А и В называют независимыми, если вероятность появления события А не меняет вероятности появления события В.

В нашей задаче так и есть: неразумный паук сворачивает налево или направо случайным образом, независимо от того, что он делал до этого.

Для нескольких независимых событий вероятность того, что все они произойдут, равна произведению вероятностей.

13. (А) Два грузовика, работая совместно, вывозят снег с улицы Нижняя Подгорная, причем первый грузовик должен сделать три рейса с грузом снега, а второй — два. Вероятность застрять с грузом снега при подъеме в горку равна 0,2 для первого грузовика и 0,25 — для второго. С какой вероятностью грузовики вывезут снег с улицы Нижняя Подгорная, ни разу не застряв на горке?

Вероятность для первого грузовика благополучно одолеть горку 1 - 0,2 = 0,8. Для второго 1 - 0,25 = 0,75. Поскольку первый грузовик должен сделать 3 рейса, а второй – два, грузовики ни разу не застрянут на горке с вероятностью 0,8cdot0,75cdot0,8cdot0,75cdot 0,8 =0,36cdot0,8=0,288.

14. Агрофирма закупает куриные яйца в двух домашних хозяйствах. 40% яиц из первого хозяйства — яйца высшей категории, а из второго хозяйства — 20% яиц высшей категории. Всего высшую категорию получает 35% яиц. Найдите вероятность того, что яйцо, купленное у этой агрофирмы, окажется из первого хозяйства.

Нарисуем все возможные исходы ситуации. Покупатель пришел в магазин, который принадлежит агрофирме, и купил яйцо. Надо найти вероятность того, что это яйцо из первого хозяйства.

Яйца могут быть только или из первого домашнего хозяйства, или из второго, причем эти два события несовместны. Других яиц в этот магазин не поступает.

Пусть вероятность того, что купленное яйцо из первого хозяйства, равна x. Тогда вероятность того, что яйцо из второго хозяйства (противоположного события), равна 1-x.

Яйца могут быть высшей категории и не высшей.
В первом хозяйстве 40% яиц имеют высшую категорию, а 60% — не высшую. Это значит, что случайно выбранное яйцо из первого хозяйства с вероятностью 40% будет высшей категории.

Во втором хозяйстве 20% яиц высшей категории, а 80% — не высшей.

Пусть случайно выбранное в магазине яйцо — из первого хозяйства и высшей категории. Вероятность этого события равна произведению вероятностей: 0,4 x.

Вероятность того, что яйцо из второго хозяйства и высшей категории, равна 0,2 (1-x).

Если мы сложим эти две вероятности, мы получим вероятность того, что яйцо имеет высшую категорию. По условию, высшую категорию имеют 35% яиц, значит, эта вероятность равна 0,35.

Мы получили уравнение:

0,4 x + 0,2 (1-x) = 0,35.

Решаем это уравнение и находим, что x = 0,75 – вероятность того, что яйцо, купленное у этой агрофирмы, оказалось из первого хозяйства.

15. Всем пациентам с подозрением на гепатит делают анализ крови. Если анализ выявляет гепатит, то результат анализа называется положительным. У больных гепатитом пациентов анализ даёт положительный результат с вероятностью 0,9. Если пациент не болен гепатитом, то анализ может дать ложный положительный результат с вероятностью 0,01. Известно, что 5% пациентов, поступающих с подозрением на гепатит, действительно больны гепатитом. Найдите вероятность того, что результат анализа у пациента, поступившего в клинику с подозрением на гепатит, будет положительным.

С чем пришел пациент в клинику? – С подозрением на гепатит. Возможно, он действительно болен гепатитом, а возможно, у его плохого самочувствия другая причина. Может быть, он просто съел что-нибудь. Вероятность того, что он болен гепатитом, равна 0,05 (то есть 5%). Вероятность того, что он здоров, равна 0,95 (то есть 95%).

Пациенту делают анализ. Покажем на схеме все возможные исходы:

Если он болен гепатитом, анализ дает положительный результат с вероятностью 0,9. То есть анализ покажет: «есть гепатит».
Заметим, что анализ не во всех случаях выявляет гепатит у того, кто действительно им болен. С вероятностью 0,1 анализ не распознает гепатит у больного.

Более того. Анализ может ошибочно дать положительный результат у того, кто не болеет гепатитом. Вероятность такого ложного положительного результата 0,01. Тогда с вероятностью 0,99 анализ даст отрицательный результат, если человек здоров.

Найдем вероятность того, что результат анализа у пациента, поступившего в клинику с подозрением на гепатит, будет положительным.

Благоприятные для этой ситуации исходы: человек болен, и анализ положительный (вероятность одновременного наступления этих двух событий равна 0,05cdot0,9 ), или человек здоров, и анализ ложный положительный (вероятность одновременного наступления этих двух событий равна 0,95cdot0,01 ). Так как события «человек болен» и «человек не болен» несовместны, то вероятность того, что результат анализа будет положительным, равна 0,05cdot0,9+0,95cdot0,01=0,0545.

Ответ: 0,0545.

16. Чтобы поступить в институт на специальность «Лингвистика», абитуриент З. должен набрать на ЕГЭ не менее 70 баллов по каждому из трёх предметов — математика, русский язык и иностранный язык. Чтобы поступить на специальность «Коммерция», нужно набрать не менее 70 баллов по каждому из трёх предметов — математика, русский язык и обществознание.

Вероятность того, что абитуриент З. получит не менее 70 баллов по математике, равна 0,6, по русскому языку — 0,8, по иностранному языку — 0,7 и по обществознанию — 0,5.
Найдите вероятность того, что З. сможет поступить хотя бы на одну из двух упомянутых специальностей.

Заметим, что в задаче не спрашивается, будет ли абитуриент по фамилии З. учиться и лингвистике, и коммерции сразу и получать два диплома. Здесь надо найти вероятность того, что З. сможет поступить хотя бы на одну из двух данных специальностей – то есть наберет необходимое количество баллов.
Для того чтобы поступить хотя бы на одну из двух специальностей, З. должен набрать не менее 70 баллов по математике. И по русскому. И еще – обществознание или иностранный.
Вероятность набрать 70 баллов по математике для него равна 0,6.
Вероятность набрать баллы по математике и русскому равна 0,6 cdot 0,8.

Разберемся с иностранным и обществознанием. Нам подходят варианты, когда абитуриент набрал баллы по обществознанию, по иностранному или по обоим. Не подходит вариант, когда ни по языку, ни по «обществу» он не набрал баллов. Значит, вероятность сдать обществознание или иностранный не ниже чем на 70 баллов равна
1 - 0,5 cdot 0,3.
В результате вероятность сдать математику, русский и обществознание или иностранный равна 0,6 cdot 0,8 cdot (1 - 0,5 cdot 0,3) = 0,408. Это ответ.

Чтобы полностью освоить тему, смотрите видеокурс по теории вероятностей. Это бесплатно.

Еще задачи ЕГЭ по теме «Теория вероятностей».

Смотрите также: парадокс Монти Холла.

Благодарим за то, что пользуйтесь нашими публикациями.
Информация на странице «Задание 3. Теория вероятностей на ЕГЭ по математике.» подготовлена нашими авторами специально, чтобы помочь вам в освоении предмета и подготовке к ЕГЭ и ОГЭ.
Чтобы успешно сдать нужные и поступить в ВУЗ или колледж нужно использовать все инструменты: учеба, контрольные, олимпиады, онлайн-лекции, видеоуроки, сборники заданий.
Также вы можете воспользоваться другими материалами из данного раздела.

Публикация обновлена:
09.03.2023

Решение типовых задач ЕГЭ по математике (профильная).

Теория вероятности.

№1.В случайном эксперименте бросают две игральные
кости.Найдите вероятность того ,что всумме выпадет 5 очков.Результат округлите
до сотых.

Решение:Всего вариантов выпадения для 2 кубиков m=62=36(каждый
из кубиков имеет 6 граней).А подходящих для нас (сумма  равна 5) всего
n=4

5=1+4=2+3=3+2=4+1

Искомая вероятность
равна Р=4
/36=0,11

Ответ:0,11

№2.В случайном эксперименте бросают три
игральные кости. Найдите вероятность того, что в сумме выпадет 16 очков.
Результат округлите до сотых.

Решение: Всего вариантов выпадения для трёх кубиков
m= 6³ = 216 (каждый из кубиков имеет 6 граней).

А подходящих для нас
(сумма равна 16) всего n= 6:

16 = 6+6+4 = 6+4+6 =
4+6+6 = 5+5+6 = 5+6+5 = 6+5+5.

Искомая вероятность
равна Р = 6/216 = ¹⁄₃₆ ≈ 0,03.

Ответ: 0,03

№3.В чемпионате по гимнастике участвуют 20
спортсменок: 8 из России, 7 из США, остальные — из Китая. Порядок, в котором
выступают гимнастки, определяется жребием. Найдите вероятность того, что спортсменка,
выступающая первой, окажется из Китая.

Решение:В чемпионате принимает
участие 20 − (8 + 7) = 5 спортсменок из Китая. Тогда вероятность того, что
спортсменка, выступающая первой, окажется из Китая, равна

Ответ: 0,25.

№ 4: В среднем из 1000 садовых насосов, поступивших
в продажу, 5 подтекают. Найдите вероятность того, что один случайно выбранный
для контроля насос не подтекает.

Решение: n= 1000 – 5 = 995 – насосов не
подтекают.  m=1000.

Вероятность того, что один случайно выбранный для
контроля насос не подтекает, равна 

  Р= n/m=995/1000 = 0,995.

Ответ : 0,995

№5:  Фабрика выпускает сумки. В среднем на 100
качественных сумок приходится восемь сумок со скрытыми дефектами. Найдите
вероятность того, что купленная сумка окажется качественной. Результат
округлите до сотых.

Решение: m= 100 + 8 = 108 – сумок всего
(качественных и со скрытыми дефектами) ; благоприятных исходов n = 100/

Вероятность того, что купленная сумка окажется
качественной, равна       Р = n/m =100/108 = 0,(925) ≈ 0,93.

Ответ : 0,93

№6 .В соревнованиях по
толканию ядра участвуют 4 спортсмена из Финляндии, 7 спортсменов из Дании, 9
спортсменов из Швеции и 5 − из Норвегии. Порядок, в котором выступают
спортсмены, определяется жребием. Найдите вероятность того, что спортсмен,
который выступает последним, окажется из Швеции.

 Решение : Всего участвует m= 4 + 7 + 9 + 5 = 25
спортсменов; благоприятных исходов n =9.

 Вероятность того, что спортсмен, который
выступает последним, окажется из Швеции, равна 

 Р = n/m =9/25 = 36/100 = 0,36.

Ответ: 0,36

№7: Научная конференция проводится в 5 дней. Всего
запланировано 75 докладов − первые три дня по 17 докладов, остальные
распределены поровну между четвертым и пятым днями. Порядок докладов
определяется жеребьёвкой. Какова вероятность, что доклад профессора
Н. окажется
запланированным на последний день конференции?

Решение: В последний день конференции
запланировано

n=(75 – 17 × 3) : 2 = 12 докладов; всего
возможных выборов m=75.

Вероятность того, что доклад профессора Н. окажется
запланированным на последний день конференции, равна Р= n/m= 12/75 = 4/25 =
0,16.

Ответ: 0,16

№8: Конкурс исполнителей проводится в 5 дней. Всего
заявлено 80 выступлений − по одному от каждой страны. В первый день 8
выступлений, остальные распределены поровну между оставшимися днями. Порядок
выступлений определяется жеребьёвкой. Какова вероятность, что выступление
представителя России состоится в третий день конкурса?

Решение: В третий день конкурса запланировано

 n=(80 – 8) : 4 = 18 выступлений ; всего
возможных выборов m=80.

Вероятность того, что выступление представителя
России состоится в третий день конкурса, равна 

Р = n/m =18/80 = 9/40 = 225/1000 = 0,225.

Ответ: 0,225.

№9 : На семинар приехали 3 ученых из Норвегии, 3 из
России и 4 из Испании. Порядок докладов определяется жеребьёвкой. Найдите
вероятность того, что восьмым окажется доклад ученого из России.

Решение: Всего участвует m= 3 + 3 + 4 = 10 ученых,
из России n=3

Вероятность того, что восьмым окажется доклад
ученого из России, равна   Р = m/n= 3/10 = 0,3.

Ответ: 0,3.

 №10: Перед началом первого тура чемпионата по
бадминтону участников разбивают на игровые пары случайным образом с помощью
жребия. Всего в чемпионате участвует 26 бадминтонистов, среди которых 10
участников из России, в том числе Руслан Орлов. Найдите вероятность того, что в
первом туре Руслан Орлов будет играть с каким-либо бадминтонистом из России?

Решение: Нужно учесть, что Руслан Орлов не может
играть сам с собою, поэтому m =25 , сам   Руслан Орлов тоже из России , значит
n =9.

Вероятность того, что в первом туре Руслан Орлов
будет играть с каким-либо бадминтонистом из России, равна    Р = m/n=   9/25 =
36/100 = 0,36.

Ответ: 0,36.

 №11: В сборнике билетов по биологии всего 55 билетов,
в 11 из них встречается вопрос по ботанике. Найдите вероятность того, что в
случайно выбранном на экзамене билете школьнику достанется вопрос по ботанике.

Решение:  Вероятность того, что в случайно
выбранном на экзамене билете школьнику достанется вопрос по ботанике, равна  Р
= m/n=  11/55 =1/5 = 0,2.

Ответ: 0,2.

№12 : В сборнике билетов по математике всего 25
билетов, в 10 из них встречается вопрос по неравенствам. Найдите вероятность
того, что в случайно выбранном на экзамене билете школьнику не достанется
вопроса по неравенствам.

Решение: Благоприятных исходов  n=25 – 10 = 15 –
билетов не содержат вопрос по неравенствам.

Вероятность того, что в случайно выбранном на
экзамене билете школьнику не достанется вопроса по неравенствам, равна   Р =
m/n= 15/25 = 3/5  = 0,6.

Ответ: 0,6

13. Две фабрики выпускают
одинаковые стекла для автомобильных фар. Первая фабрика выпускает 45% этих
стекол, вторая — 55%. Первая фабрика выпускает 3% бракованных стекол, а вторая
— 1%. Найдите вероятность того, что случайно купленное в магазине стекло
окажется бракованным.

Решение.Вероятность того, что стекло куплено на
первой фабрике и оно бракованное: 0,45 • 0,03 = 0,0135.

Вероятность того, что стекло куплено на второй
фабрике и оно бракованное: 0,55 • 0,01 = 0,0055.

Поэтому по формуле полной вероятности вероятность
того, что случайно купленное в магазине стекло окажется бракованным
равна                            0,0135 + 0,0055 = 0,019.

Ответ: 0,019

14. Если гроссмейстер А.
играет белыми, то он выигрывает у гроссмейстера Б. с вероятностью 0,52. Если А.
играет черными, то А. выигрывает у Б. с вероятностью 0,3. Гроссмейстеры А. и Б.
играют две партии, причем во второй партии меняют цвет фигур. Найдите
вероятность того, что А. выиграет оба раза.

Решение.Возможность выиграть первую и вторую
партию не зависят друг от друга. Вероятность произведения независимых событий
равна произведению их вероятностей: 0,52 · 0,3 = 0,156.

 Ответ: 0,156.

15:  Вася, Петя, Коля и Лёша
бросили жребий — кому начинать игру. Найдите вероятность того, что начинать
игру должен будет Петя.

Решение: Жребий начать игру может выпасть каждому
из четырех мальчиков , значит m=4. Вероятность того, что это будет именно
Петя        Р = m/n= 1/4 = 0,25

 Ответ: 0,25.

 №16: В чемпионате мира
участвует 20 команд. С помощью жребия их нужно разделить на пять групп по
четыре команды в каждой. В ящике вперемешку лежат карточки с номерами
групп:     

1, 1, 1, 1, 2, 2, 2, 2, 3, 3, 3, 3, 4, 4, 4, 4,
5, 5, 5, 5.

Капитаны команд тянут по одной карточке. Какова
вероятность того, что команда России окажется в третьей
группе.   

Решение: Всего команд 20, значит возможных
вариантов m =20 . Благоприятных исходов  n =4 ( четыре карточки с  цифрой 3) .
Вероятность выпадения нужного исхода Р = n/m=  4/20 = 0,2.

Ответ: 0,2.

№17.На экзамене по геометрии
школьник отвечает на один вопрос из списка экзаменационных вопросов.
Вероятность того, что это вопрос по теме «Внешние углы», равна 0,2. Вероятность
того, что это вопрос по теме «Тригонометрия», равна 0,25. Вопросов, которые
одновременно относятся к этим двум темам, нет. Найдите вероятность того, что на
экзамене школьнику достанется вопрос по одной из этих двух тем.

Решение.Введем два события:А:
выбор вопроса по теме «Внешние углы»;B: выбор вопроса по теме
«Тригонометрия».Вероятности этих событий:

Так как вопросов,
которые одновременно относятся к этим двум темам, нет, то события несовместны и
вероятность их суммы можно вычислить по формуле:

Ответ: 0,45

№18: В торговом центре два
одинаковых автомата продают кофе. Вероятность того, что к концу дня в автомате
закончится кофе, равна 0,3. Вероятность того, что кофе закончится в обоих
автоматах, равна 0,12. Найдите вероятность того, что к концу дня кофе останется
в обоих автоматах.

Решение: Рассмотрим события :А = кофе закончится в
первом автомате, В = кофе закончится во втором автомате.  Тогда A•B = кофе
закончится в обоих автоматах,  A + B = кофе закончится хотя бы в одном
автомате. По условию P(A) = P(B) = 0,3; P(A•B) = 0,12. События A и B
совместные, вероятность суммы двух совместных событий равна сумме вероятностей
этих событий, уменьшенной на вероятность их произведения:

P(A + B) = P(A) +
P(B) − P(A•B) = 0,3 + 0,3 − 0,12 = 0,48. Следовательно, вероятность
противоположного события, состоящего в том, что кофе останется в обоих
автоматах, равна 1 − 0,48 = 0,52.

Ответ: 0,52.

19:Биатлонист пять раз стреляет
по мишеням. Вероятность попадания в мишень при одном выстреле равна 0,8.
Найдите вероятность того, что биатлонист первые три раза попал в мишени, а
последние два раза промахнулся. Результат округлите до сотых.

Решение: Результат каждого
следующего выстрела не зависит от предыдущих. Поэтому события «попал при первом
выстреле», «попал при втором выстреле» и т.д. независимы.Вероятность каждого
попадания равна 0,8. Значит, вероятность промаха равна 1 – 0,8 = 0,2.
1 выстрел: Р= 0,8 ;  2
выстрел : Р= 0,8 ; 3 выстрел : Р= 0,8;4 выстрел :Р = 0,2  ;5 выстрел :Р= 0,2
.По формуле умножения
вероятностей независимых событий, получаем, что искомая вероятность равна:

 Р=0,8 ∙ 0,8 ∙
0,8 ∙ 0,2 ∙ 0,2 = 0,02048 ≈ 0,02.

Ответ: 0,02.

№ 20. В магазине стоят два
платёжных автомата. Каждый из них может быть неисправен с вероятностью 0,05
независимо от другого автомата. Найдите вероятность того, что хотя бы один
автомат исправен.

Решение.Найдем вероятность того,
что неисправны оба автомата. Эти события независимые, вероятность их
произведения равна произведению вероятностей этих событий: 0,05 • 0,05 =
0,0025. Событие, состоящее в том, что исправен хотя бы один автомат,
противоположное. Следовательно, его вероятность равна 1 − 0,0025 = 0,9975.

Ответ: 0,9975.

№ 21. Помещение освещается
фонарём с двумя лампами. Вероятность перегорания одной лампы в течение года
равна 0,3. Найдите вероятность того, что в течение года хотя бы одна лампа не
перегорит.

Решение.Найдем вероятность того,
что перегорят обе лампы. Эти события независимые, вероятность их произведения
равно произведению вероятностей этих событий: 0,3·0,3 = 0,09. Событие,
состоящее в том, что не перегорит хотя бы одна лампа, противоположное.
Следовательно, его вероятность равна 1 − 0,09 = 0,91.

Ответ: 0,91.

№22: Вероятность того, что
новый электрический чайник прослужит больше года, равна 0,97. Вероятность того,
что он прослужит больше двух лет, равна 0,89. Найдите вероятность того, что он
прослужит меньше двух лет, но больше года.

Решение .Пусть  A = «чайник
прослужит больше года, но меньше двух лет», В = «чайник прослужит больше двух
лет», тогда A + B = «чайник прослужит больше года». События A и В совместные,
вероятность их суммы равна сумме вероятностей этих событий, уменьшенной на
вероятность их произведения. Вероятность произведения этих событий, состоящего
в том, что чайник выйдет из строя ровно через два года — строго в тот же день,
час и секунду — равна нулю. Тогда: P(A + B) = P(A) + P(B) − P(A•B) = P(A) +
P(B), откуда, используя данные из условия, получаем 0,97 = P(A) + 0,89.Тем
самым, для искомой вероятности имеем: P(A) = 0,97 − 0,89 = 0,08.

Ответ: 0,08.

№24: Агрофирма закупает
куриные яйца в двух домашних хозяйствах. 40% яиц из первого хозяйства — яйца
высшей категории, а из второго хозяйства — 20% яиц высшей категории. Всего
высшую категорию получает 35% яиц. Найдите вероятность того, что яйцо,
купленное у этой агрофирмы, окажется из первого хозяйства.

Решение. Пусть в первом хозяйстве
агрофирма закупает x  яиц, в том числе, 0.4x яиц высшей категории, а во втором
хозяйстве y—  яиц, в том числе  02y яиц высшей категории. Тем самым, всего
агрофирма закупает  x+y яиц, в том числе 0.4x +0.2y  яиц высшей категории. По
условию, высшую категорию имеют 35% яиц, тогда:  (0.4x+0.2y)/(x+y) =0.35  ,
0.4x+0.2y=0.35(x+y) , 0.05x=0.15y , x=3y.Следовательно, у первого хозяйства
закупают в три раза больше яиц, чем у второго. Поэтому вероятность того, что
купленное яйцо окажется из первого хозяйства равна
Р=3y/(3y+y) =3/4= 0.75

Ответ : 0,75

№24: На клавиатуре телефона 10
цифр, от 0 до 9. Какова вероятность того, что случайно нажатая цифра будет
чётной?

Решение.На клавиатуре телефона 
m=10 цифр, из них  n=5 четных: 0, 2, 4, 6, 8. Поэтому вероятность того, что
случайно будет нажата четная цифра равна  Р = n/m=5 / 10 = 0,5.

Ответ: 0,5.

№25: Какова вероятность того,
что случайно выбранное натуральное число от 10 до 19 делится на три?

Решение.Натуральных чисел от 10 до
19  m=10, из них на три делятся три числа: n= 12, 15, 18. Следовательно,
искомая вероятность равна   Р = n/m=3/10 = 0,3.

Ответ: 0,3.

№26.Ковбой
Джон попадает в муху на стене с вероятностью 0,9, если стреляет из
пристрелянного револьвера. Если Джон стреляет из непристрелянного револьвера,
то он попадает в муху с вероятностью 0,4. На столе лежат 10 револьверов, из них
только 4 пристрелянные. Ковбой Джон видит на стене муху, наудачу хватает первый
попавшийся револьвер и стреляет в муху. Найдите вероятность того, что Джон
промахнётся.

Решение.Ковбой Джон может наудачу
схватить как пристрелляный, так и не пристрелянный револьвер. Так как на столе
10 револьверов и из них только 4 пристрелянные, то вероятность выбора
пристрелянного револьвера равна 
4/10=0,4,а
непристрелянного 1-0,4=0,6.Известно, что если он выстреливает из пристрелянного
револьвера, то попадает в цель с вероятностью 0,9, значит, вероятность такого
события будет равна0,4*0,9=0,36,а вероятность выбора непристрелянного
револьвера и попадания из него в цель, равна 0,6*0,4=0,24.Если произойдет или
первое или второе событие, то Ковбой Джон попадет в цель и вероятность этого события
равна 0,36+0,24=0,6,тогда вероятность промаха 1-0,6=0,4.

Ответ: 0,4.

№27: В группе туристов 5
человек. С помощью жребия они выбирают двух человек, которые должны идти в село
за продуктами. Турист А. хотел бы сходить в магазин, но он подчиняется жребию.
Какова вероятность того, что А. пойдёт в магазин?

Решение.Всего туристов  m=5,
случайным образом из них выбирают n=2. Вероятность быть выбранным равна Р =
n/m=2 / 5 = 0,4.

Ответ: 0,4.

№28: Перед началом
футбольного матча судья бросает монетку, чтобы определить, какая из команд
начнёт игру с мячом. Команда «Физик» играет три матча с разными командами.
Найдите вероятность того, что в этих играх «Физик» выиграет жребий ровно два
раза.

Решение.Обозначим «Р» ту сторону
монеты, которая отвечает за выигрыш жребия «Физиком», другую сторону монеты
обозначим «0». Тогда благоприятных комбинаций  n=3: РР0, Р0Р, 0РР, а всего
комбинаций  m=2
3 = 8:Тем самым, искомая вероятность равна:  Р=n/m=3/8= 0,375.

Ответ: 0,375.

29:  В классе 26 человек,
среди них два близнеца — Андрей и Сергей. Класс случайным образом делят на две
группы по 13 человек в каждой. Найдите вероятность того, что Андрей и Сергей
окажутся в одной группе.

Решение.Пусть один из близнецов
находится в некоторой группе. Вместе с ним в группе может оказаться n=12
человек из m=25 оставшихся одноклассников. Вероятность этого события
равнаP=n/m= 12 / 25 = 0,48.

Ответ : 0,4

№30.В
случайном эксперименте симметричную монету бросают трижды. Найдите вероятность
того, что орел выпадет ровно два раза.

Решение.Обозначим выпадение орла буквой О, а
выпадение решки буквой Р. Возможных восемь исходов:OOO,  OОР,  
ОРО,   ОРР,   РОО,   РОР,  РРО,  
РРР

Из них благоприятными являются OОР, ОРО и РОО.
Поэтому искомая вероятность равна  то есть 0,375. (Этот
подход затруднителен в случае большого числа бросаний монетки.)

 Ответ: 0,375.

№31 Ковбой
Джон попадает в муху на стене с вероятностью 0,9, если стреляет из
пристрелянного револьвера. Если Джон стреляет из непристрелянного револьвера,
то он попадает в муху с вероятностью 0,2. На столе лежит 10 револьверов, из них
только 4 пристрелянные. Ковбой Джон видит на стене муху, наудачу хватает первый
попавшийся револьвер и стреляет в муху. Найдите вероятность того, что Джон
промахнётся.

Решение.Запишем,
как могло случиться, что «Джон промахнулся». «Ковбой схватил
пристрелянный револьвер И не попал в муху, ИЛИ ковбой схватил непристрелянный
револьвер И не попал в муху.» Сначала разберемся с пистолетами: —
Вероятность схватить пристрелянный пистолет равна 4/10 = 0,4. Мы вычислили её
по определению вероятности: здесь один пистолет = одно элементарное событие,
один пристрелянный пистолет = одно благоприятствующее событие. — Вероятность
схватить непристрелянный пистолет равна (10−4)/10 = 0,6. Вычислили аналогично,
определив число непристрелянных пистолетов. Затем разберемся с мухой: — Если
ковбой стрелял из пристрелянного револьвера, то он НЕ попал в муху с
вероятностью 1−0,9=0,1. — Если ковбой стрелял из непристрелянного револьвера,
то он НЕ попал в муху с вероятностью 1−0,2=0,8. Здесь мы воспользовались
формулой для вероятности противоположного события, потому что в условии даны
вероятности попадания в муху из разных пистолетов, но не промахов. Теперь
вернемся к нашей формулировке события «Ковбой схватил…» и вместо
текста, описывающего составляющие события, подставим полученные числа — их
вероятности, а вместо союзов «И» и «ИЛИ» знаки «·» и
«+» соответственно. Получаем:

0,4·0,1 + 0,6·0,8 = 0,04 + 0,48 = 0,52.

Ответ :0,52

№32 В
группе туристов 5 человек. С помощью жребия они выбирают двух человек, которые
должны идти в село за продуктами. Турист А. хотел бы сходить в магазин, но он
подчиняется жребию. Какова вероятность того, что А. пойдёт в магазин?

Решение.

Всего туристов  m=5, случайным образом из них
выбирают n=2. Вероятность быть выбранным равна Р = n/m=2 / 5 = 0,4.

Ответ: 0,4.

№33 Перед
началом футбольного матча судья бросает монетку, чтобы определить, какая из
команд начнёт игру с мячом. Команда «Физик» играет три матча с разными
командами. Найдите вероятность того, что в этих играх «Физик» выиграет жребий
ровно два раза.

Решение.

Обозначим «Р» ту сторону монеты, которая отвечает
за выигрыш жребия «Физиком», другую сторону монеты обозначим «0». Тогда
благоприятных комбинаций  n=3: РР0, Р0Р, 0РР, а всего комбинаций  m=2^3 = 8:Тем
самым, искомая вероятность равна:                     Р=n/m=3/8= 0,375.

Ответ: 0,375.

№34 На
рок-фестивале выступают группы — по одной от каждой из заявленных стран.
Порядок выступления определяется жребием. Какова вероятность того, что группа
из Дании будет выступать после группы из Швеции и после группы из Норвегии?
Результат округлите до сотых.

Решение.

Общее количество выступающих на фестивале групп
для ответа на вопрос неважно. Сколько бы их ни было, для указанных стран есть 6
способов взаимного расположения среди выступающих (Д — Дания, Ш — Швеция, Н —
Норвегия):

m=…Д…Ш…Н…, …Д…Н…Ш…,
…Ш…Н…Д…, …Ш…Д…Н…, …Н…Д…Ш…, …Н…Ш…Д… = 6

Дания находится после Швеции и Норвегии n=2.

 Поэтому вероятность того, что группы случайным
образом будут распределены именно так, равна  Р = n/m=2/6=0,333… = 0,33.

Ответ: 0,33

№35 В
классе 26 человек, среди них два близнеца — Андрей и Сергей. Класс случайным
образом делят на две группы по 13 человек в каждой. Найдите вероятность того,
что Андрей и Сергей окажутся в одной группе.

Решение.

Пусть один из близнецов находится в некоторой
группе. Вместе с ним в группе может оказаться n=12 человек из m=25 оставшихся
одноклассников. Вероятность этого события равнаP=n/m= 12 / 25 = 0,48.

Ответ : 0,48

№36 Чтобы
поступить в институт на специальность «Лингвистика», абитуриент должен набрать
на ЕГЭ не менее 70 баллов по каждому из трёх предметов — математика, русский
язык и иностранный язык. Чтобы поступить на специальность «Коммерция», нужно
набрать не менее 70 баллов по каждому из трёх предметов — математика, русский
язык и обществознание. Вероятность того, что абитуриент З. получит не менее 70
баллов по математике, равна 0,6, по русскому языку — 0,8, по иностранному языку
— 0,7 и по обществознанию — 0,5.

Найдите вероятность того, что З. сможет поступить
хотя бы на одну из двух упомянутых специальностей.

Решение.

Для того, чтобы поступить хоть куда-нибудь, З.
нужно сдать и русский, и математику как минимум на 70 баллов, а помимо этого
еще сдать иностранный язык или обществознание не менее, чем на 70 баллов. Пусть
A, B, C и D — это события, в которых З. сдает соответственно математику,
русский, иностранный и обществознание не менее, чем на 70 баллов.    
Р=0,6*0,8*(0,7+0,5-0,7*0,5)=0,408

Ответ: 0,408.

№37 Из
районного центра в деревню ежедневно ходит автобус. Вероятность того, что в
понедельник в автобусе окажется меньше 20 пассажиров, равна 0,94. Вероятность
того, что окажется меньше 15 пассажиров, равна 0,56. Найдите вероятность того,
что число пассажиров будет от 15 до 19.

Решение.

Рассмотрим события A = «в автобусе меньше 15
пассажиров»     и                           В = «в автобусе от 15 до 19
пассажиров». Их сумма — событие A + B = «в автобусе меньше 20 пассажиров».
События A и В несовместные, вероятность их суммы равна сумме вероятностей этих
событий:

P(A + B) = P(A) + P(B).

Тогда, используя данные задачи, получаем: 0,94 =
0,56 + P(В), откуда P(В) = 0,94 − 0,56 = 0,38.

Ответ: 0,38.

№38 В
Волшебной стране бывает два типа погоды: хорошая и отличная, причём погода,
установившись утром, держится неизменной весь день. Известно, что с
вероятностью 0,8 погода завтра будет такой же, как и сегодня. Сегодня 3 июля,
погода в Волшебной стране хорошая. Найдите вероятность того, что 6 июля в
Волшебной стране будет отличная погода.

Решение.

Для погоды на 4, 5 и 6 июля есть 4 варианта: ХХО,
ХОО, ОХО, ООО (здесь Х — хорошая, О — отличная погода). Найдем вероятности
наступления такой погоды:

P(XXO) = 0,8•0,8•0,2 = 0,128;

P(XOO) = 0,8•0,2•0,8 = 0,128;

P(OXO) = 0,2•0,2•0,2 = 0,008;

P(OOO) = 0,2•0,8•0,8 = 0,128.

Указанные события несовместные, вероятность их
сумы равна сумме вероятностей этих событий:

P(ХХО) + P(ХОО) + P(ОХО) + P(ООО) = 0,128 + 0,128
+ 0,008 + 0,128 = 0,392.

Ответ: 0,392.

№39 Вероятность
того, что батарейка бракованная, равна 0,06. Покупатель в магазине выбирает
случайную упаковку, в которой две таких батарейки. Найдите вероятность того,
что обе батарейки окажутся исправными.

Решение.

Вероятность того, что батарейка исправна, равна
0,94. Вероятность произведения независимых событий (обе батарейки окажутся
исправными) равна произведению вероятностей этих
событий:                          Р= 0,94·0,94 = 0,8836.

Ответ: 0,8836. 

№40 Перед
началом волейбольного матча капитаны команд тянут честный жребий, чтобы
определить, какая из команд начнёт игру с мячом. Команда «Статор» по очереди
играет с командами «Ротор», «Мотор» и «Стартер». Найдите вероятность того, что
«Статор» будет начинать только первую и последнюю игры.

Решение.

Требуется найти вероятность произведения трех
событий: «Статор» начинает первую игру, не начинает вторую игру, начинает
третью игру. Вероятность произведения независимых событий равна произведению
вероятностей этих событий. Вероятность каждого из них равна 0,5, откуда
находим: 0,5•0,5•0,5 = 0,125.

Ответ: 0,125.

Примеры задач по теории вероятности

В данном разделе размещены задачи с решениями по теории вероятностей из разных разделов, которые изучаются студентами и даже школьниками. Помимо подробных решений типовых задач на сайте есть онлайн-учебник, онлайн-калькуляторы по теории вероятностей, формулы, списки учебников и многое другое (см. ссылки ниже).

Если у вас есть трудности с решением заданий по терверу, обращайтесь, мы готовы помочь. Стоимость консультации от 70 рублей, срок от 1 дня, оформление в Word. Подробнее тут: Теория вероятностей для студентов.

Каталог решенных задач

Комбинаторика

  • Элементы комбинаторики (10 задач)
  • Формула включений-исключений (6 задач)

Случайные события

  • Классическое определение вероятности (13 задач)
  • Геометрическое определение вероятности (7 задачи)
  • Формула Бернулли (8 задач)
  • Теоремы сложения и умножения вероятностей (11 задач)
  • Формула полной вероятности и формула Байеса (12 задач)
  • Локальная и интегральная теоремы Лапласа (10 задач)
  • Отклонение частоты от вероятности (8 задач)

Случайные величины

  • Дискретная случайная величина (12 задач)
    • Табличный закон распределения (10 задач)
    • Биномиальный закон распределения (10 задач)
    • Гипергеометрический закон распределения (10 задач)
    • Геометрический закон распределения (5 задач)
    • Закон распределения Пуассона (8 задач)
  • Двумерная дискретная случайная величина (10 задач)
  • Непрерывная случайная величина (12 задач)
    • Нормальная случайная величина (11 задач)
    • Равномерная случайная величина (8 задач)
    • Показательная случайная величина (10 задач)
  • Двумерная непрерывная случайная величина (8 задач)
  • Неравенства Маркова и Чёбышева, ЗБЧ (10 задач)
  • Производящая функция (5 задач)
  • Характеристическая функция (5 задач)

Случайные функции

  • Случайные процессы (7 задач)
  • Стационарные случайные процессы (6 задач)
  • Цепи Маркова (6 задач)

Консультируем по задачам теории вероятностей

Контрольные по вузам

  • ИДЗ по ТВиМС, МЭСИ
  • Контрольная по ТВ, ВЗФЭИ (ФУ)
  • Теория вероятностей, МАИ
  • Типовой расчет по теории вероятностей, МИРЭА

Решебник по теории вероятности

Больше 10000 решенных и оформленных задач по всем темам теории вероятности в решебнике:

Полезные ссылки для изучения ТВ

  • Онлайн калькуляторы
  • Онлайн учебник
  • Формулы и таблицы
  • Решенные контрольные
  • Что такое ТВиМС
  • Сдача тестов
  • Решение на заказ
  • Онлайн помощь

На чтение 16 мин Просмотров 105к. Опубликовано 25 мая, 2018

Вероятность — очень лёгкая тема, если концентрироваться на смысле задач, а не на формулах. Найти вероятность того что — не просто. И  как решать задачи на вероятность?. Во-первых, что такое вероятность? Это шанс, что какое-то событие произойдёт. Если мы говорим, что вероятность некоторого события 50%, что это значит? Что оно либо произойдет, либо не произойдет — одно из двух. Таким образом подсчитать значение вероятности очень просто — нужно взять количество подходящих нам вариантов и разделить на количество всех возможных вариантов. Например, шанс получить решку при подбрасывании монеты это ½. Как мы получаем ½? Всего у нас два возможных варианта (орёл и решка), из них нам подходит один (решка), так мы и получаем вероятность ½.

вероятность

Как мы уже с вами увидели, вероятность может быть выражена как в процентах, так и в обычных числах. Важно: на ЕГЭ вам нужно будет записать ответ в числах, не в процентах. Принято, что вероятность изменяется от 0 (никогда не произойдет) до 1 (абсолютно точно произойдет). Также можно сказать, что всегда

Вероятность подходящих событий + вероятность неподходящих событий = 1

Теперь мы точно понимаем, как считать вероятность отдельного события, и даже такие задачи есть в банке ФИПИ, но понятно, что на этом всё не заканчивается. Чтобы жизнь была веселее, в задачах на вероятность обычно происходят как минимум два события, и надо посчитать вероятность с учетом каждого из них.

Содержание

  1. Вероятность нескольких событий
  2. Задачи и решения задач на вероятность
  3. Вероятность нескольких событий
  4. Дополняющая вероятность

Вероятность нескольких событий

Подсчитываем вероятность каждого события в отдельности, затем между дробями ставим знаки:

1. Если нужно первое И второе событие, то умножаем.

2. Если нужно первое ИЛИ второе событие, то складываем.

Задачи и решения задач на вероятность

Задача 1. Среди натуральных чисел от 23 до 37 случайно выбирают одно число. Найдите вероятность того, что оно не делится на 5.

Решение:

Вероятность, это отношение благоприятных вариантов к общему их количеству.

Всего в этом промежутке 15 чисел. Из них на 5 делится всего 3, значит не делится 12.

Вероятность тогда: формула 1

Ответ: 0,8.

Задача 2. Для дежурства в столовой случайно выбирают двух учащихся класса. Какова вероятность того, что дежурить будут два мальчика, если в классе обучается 7 мальчиков и 8 девочек?

Решение: Вероятность, это отношение благоприятных вариантов к общему их количеству. В классе 7 мальчиков, это благоприятные варианты. А всего 15 учеников.

Вероятность что первый дежурный мальчик:

формула 2

Вероятность что второй дежурный мальчик:

формула 3

Раз оба должны быть мальчики, вероятности перемножим:

формула 4

Ответ: 0,2.

Задача 3. На борту самолёта 12 мест рядом с запасными выходами и 18 мест за перегородками, разделяющими салоны. Остальные места неудобны для пассажира высокого роста. Пассажир В. высокого роста. Найдите вероятность того, что на регистрации при случайном выборе места пассажиру В. достанется удобное место, если всего в самолёте 300 мест.

Решение: Пассажиру В. удобны 30 мест (12 + 18 = 30), а всего в самолете 300 мест. Поэтому вероятность того, что пассажиру В. достанется удобное место равна 30/300, т. е. 0,1.

Задача 4. В сборнике билетов по математике всего 25 билетов, в 10 из них встречается вопрос по неравенствам.

Найдите вероятность того, что в случайно выбранном на экзамене билете школьнику не достанется вопроса по неравенствам.

Решение: Из 25 билетов 15 не содержат вопроса по неравенствам, поэтому вероятность того, что в случайно выбранном на экзамене билете школьнику не достанется вопроса по неравенствам, равна 15/25, т. е. 0,6.

Задача 5. В сборнике билетов по химии всего 35 билетов, в 7 из них встречается вопрос по кислотам.

Найдите вероятность того, что в случайно выбранном на экзамене билете школьнику не достанется вопроса по кислотам.

Решение: Из 35 билетов 28 не содержат вопроса по кислотам, поэтому вероятность того, что в случайно выбранном на экзамене билете школьнику не достанется вопроса по кислотам, равна 28/35, т. е. 0,8.

Задача 6. В среднем из 500 садовых насосов, поступивших в продажу, 2 подтекают. Найдите вероятность того, что один случайно выбранный для контроля насос не подтекает.

Решение: Если из 500 насосов 2 подтекают, то 498 не подтекают. Следовательно, вероятность выбора хорошего насоса — 498/500, т. е. 0,996.

Задача 7. Вероятность того, что новый пылесос в течение года поступит в гарантийный ремонт, равна 0,065. В некотором городе из 1000 проданных пылесосов в течение года в гарантийную мастерскую поступило 70 штук.

На сколько отличается частота события «гарантийный ремонт» от его вероятности в этом городе?

Решение: Частота события «гарантийный ремонт» равна 70/1000, т. е. 0,07. Она отличается от предсказанной вероятности на 0,005 (0,07 – 0,065 = 0,005).

Задача 8. В чемпионате по гимнастике участвуют 50 спортсменок: 18 из России, 14 из Украины, остальные — из Белоруссии. Порядок, в котором выступают гимнастки, определяется жребием.

Найдите вероятность того, что спортсменка, выступающая первой, окажется из Белоруссии.

Решение: Всего участниц на чемпионате 50, а спортсменок из Белоруссии — 18 (50 – 18 – 14 = 18).

Вероятность того, что первой будет выступать спортсменка из Белоруссии — 18 из 50, т. е. 18/50, или 0,36.

Задача 9. Научная конференция проводится в 5 дней. Всего запланировано 80 докладов — первые три дня по 12 докладов, остальные распределены поровну между четвертым и пятым днями. Порядок докладов определяется жеребьёвкой.

Какова вероятность, что доклад профессора М. окажется запланированным на последний день конференции?

Решение: За первые три дня будут прочитаны 36 докладов (12 ∙ 3 = 36), на последние два дня планируется 44 доклада. Поэтому на последний день запланировано 22 докладов (44 : 2 = 22). Значит, вероятность того, что доклад профессора М. окажется запланированным на последний день конференции, равна 22/80, т. е. 0,275.

Задача 10.

Перед началом первого тура чемпионата по шахматам участников разбивают на игровые пары случайным образом с помощью жребия. Всего в чемпионате участвует 26 шахматистов, среди которых 14 участников из России, в том числе Егор Косов.

Найдите вероятность того, что в первом туре Егор Косов будет играть с каким-либо шахматистом из России?

Решение: В первом туре Егор Косов может сыграть с 25 шахматистами (26 – 1 = 25), из которых 13 ― из России. Значит, вероятность того, что в первом туре Егор Косов будет играть с каким-либо шахматистом из России, равна 13/25, или 0,52.

Задача 11.

В чемпионате мира участвуют 16 команд. С помощью жребия их нужно разделить на четыре группы по четыре команды в каждой. В ящике вперемешку лежат карточки с номерами групп: 1, 1, 1, 1, 2, 2, 2, 2, 3, 3, 3, 3, 4, 4, 4, 4.

Капитаны команд тянут по одной карточке. Какова вероятность того, что команда России окажется во второй группе?

Решение: Вероятность того, что команда России окажется во второй группе, равна отношению количества карточек с номером 2, к общему числу карточек, т. е. 4/16, или 0,25.

Задача 12.  В группе туристов 5 человек. С помощью жребия они выбирают двух человек, которые должны идти в село за продуктами. Турист А. хотел бы сходить в магазин, но он подчиняется жребию. Какова вероятность того, что А. пойдёт в магазин?

Решение: Выбирают двоих туристов из пяти. Следовательно, вероятность быть выбранным равна 2/5, т. е. 0,4.

Задача 13. В группе туристов 30 человек. Их вертолётом в несколько приёмов забрасывают в труднодоступный район по 6 человек за рейс. Порядок, в котором вертолёт перевозит туристов, случаен. Найдите вероятность того, что турист П. полетит первым рейсом вертолёта.

Решение: На первом рейсе 6 мест, всего мест 30. Тогда вероятность того, что турист полетит первым рейсом вертолёта, равна 6/30, или 0,2.

Задача 14. Какова вероятность того, что случайно выбранное натуральное число от 10 до 19 делится на три?

Решение: Натуральных чисел от 10 до 19 десять, из них на 3 делятся три числа: 12, 15 и 18. Следовательно, искомая вероятность равна 3/10, т. е. 0,3.

Вероятность нескольких событий

Задача 1. Перед началом волейбольного матча капитаны команд тянут честный жребий, чтобы определить, какая из команд начнёт игру с мячом. Команда «Стартер» по очереди играет с командами «Ротор», «Мотор» и «Стратор». Найдите вероятность того, что «Стартер» будет начинать только вторую игру.

Решение: 

Тип вопроса: совмещение событий.

Нас устроит следующий вариант: «Статор» не начинает первую игру, начинает вторую игру, не начинает третью игру. Вероятность такого развития событий равна произведению вероятностей каждого из этих событий. Вероятность каждого из них равна 0,5, следовательно: 0,5 · 0,5 · 0,5 = 0,125.

Задача 2. Чтобы пройти в следующий круг соревнований, футбольной команде нужно набрать хотя бы 4 очка в двух играх. Если команда выигрывает, она получает 3 очка, в случае ничьей ― 1 очко, если проигрывает ― 0 очков. Найдите вероятность того, что команде удастся выйти в следующий круг соревнований. Считайте, что в каждой игре вероятности выигрыша и проигрыша одинаковы и равны 0,4.

Решение: 

Тип вопроса: совмещение событий.

Задачу выполняют несколько вариантов:

Игра №1 Игра №2 Вероятность данного варианта
3 1 0,4 · 0,2 = 0,08
1 3 0,2 · 0,4 = 0,08
3 3 0,4 · 0,4 = 0,16

Вероятность происхождения какого-либо их этих 3-х вариантов равна сумме вероятностей каждого из вариантов: 0,08 + 0,08 + 0,16 = 0,32.

Задача 3. В классе учится 21 человек. Среди них две подруги: Аня и Нина. Класс случайным образом делят на 7 групп, по 3 человека в каждой. Найти вероятность того что Аня и Нина окажутся в одной группе.

Решение: 

Тип вопроса: уменьшение групп.

Вероятность попадания Ани в одну из групп равна 1. Вероятность попадания Нины в ту же группу равна 2 из 20 (2 оставшихся места в группе, а человек осталось 20). 2/20 = 1/10 = 0,1.

Задача 4. В кармане у Пети было 4 монеты по рублю и 2 монеты по два рубля. Петя, не глядя, переложил какие-то 3 монеты в другой карман. Найдите вероятность того, что обе двухрублёвые монеты лежат в одном кармане.

Решение:

Способ №1

Тип задачи: уменьшение групп.

Представим, что шесть монет делят на две группы по три монеты. Вероятность, что первая однорублевая монета попадет в один из карманов (групп) = 1.

Вероятность, что две двухрублевые монеты попадут в этот же карман = количество оставшихся мест в этом кармане/на количество оставшихся мест в обоих карманах = 2/5 = 0,4.

Способ №2

Тип вопроса: совмещение событий.

Задачу выполняют в несколько вариантов:

Если Петя переложил в другой карман три из четырех рублевых монет (а двухрублевые не перекладывал), или если переложил в другой карман обе двухрублевые монеты и одну рублевую одним из трех способов: 1, 2, 2; 2, 1, 2; 2, 2, 1. Можно изобразить это на схеме (перекладывает Петя в карман 2, поэтому будем высчитывать вероятности в колонке «карман 2»):

формула 5

Вероятность происхождения какого-либо их этих 4-х вариантов равна сумме вероятностей каждого из вариантов: формула 6

Задача 5. В кармане у Пети было 2 монеты по 5 рублей и 4 монеты по 10 рублей. Петя, не глядя, переложил какие-то 3 монеты в другой карман. Найдите вероятность того, что пятирублевые монеты лежат теперь в разных карманах.

Решение:

Тип задачи: уменьшение групп.

Способ №1

Представим, что шесть монет делят на две группы по три монеты. Вероятность, что первая двухрублевая монета попадет в один из карманов (групп) = 1. Вероятность, что вторая монета попадет в другой карман = количество оставшихся мест в другом/ на количество оставшихся мест в обоих карманах = 3/5 = 0,6.

Способ №2

Тип вопроса: совмещение событий.

Задачу выполняют несколько вариантов:

Чтобы пятирублевые монеты оказались в разных карманах, Петя должен взять из кармана одну пятирублевую и две десятирублевые монеты. Это можно сделать тремя способами: 5, 10, 10; 10, 5, 10 или 10, 10, 5. Можно изобразить это на схеме (перекладывает Петя в карман 2, поэтому будем высчитывать вероятности в колонке «карман 2»):

формула 7

Вероятность происхождения какого-либо их этих 4-х вариантов равна сумме вероятностей каждого из вариантов: формула 8

Задача 6. В случайном эксперименте симметричную монету бросают трижды. Найдите вероятность того, что орёл выпадет ровно два раза.

Решение: Тип вопроса: нахождение желаемого и действительного совмещение событий Нас устраивают три варианта:

Орёл ― решка ― орёл;

Орёл ― орёл ― решка;

Решка ― орёл ― орёл;

Вероятность каждого случая ― 1/2, а каждого варианта ― 1/8 (1/2 ∙ 1/2 ∙ 1/2 = 1/8)

Нас устроит либо первый, либо второй, либо третий вариант. Следовательно, складываем их вероятности и получаем 3/8 (1/8 + 1/8 + 1/8 = 3/8), т. е. 0,375.

Задача 7. Если гроссмейстер А. играет белыми, то он выигрывает у гроссмейстера Б. с вероятностью 0,5. Если А. играет черными, то А. выигрывает у Б. с вероятностью 0,34. Гроссмейстеры А. и Б. играют две партии, причем во второй партии меняют цвет фигур. Найдите вероятность того, что А. выиграет оба раза.

Решение: 

Тип вопроса: совмещение событий.

В любом случае А. будет играть как белыми, так и черными, поэтому нас устроит вариант, когда гроссмейстер А. выиграет, играя белыми (вероятность ― 0,5), а также играя чёрными (вероятность ― 0,34). Поэтому надо перемножить вероятности этих двух событий: 0,5 ∙ 0,34 = 0,17.

Задача 8. Вероятность того, что батарейка бракованная, равна 0,02. Покупатель в магазине выбирает случайную упаковку, в которой две таких батарейки. Найдите вероятность того, что обе батарейки окажутся исправными.

Решение: 

Тип вопроса: совмещение событий.

Вероятность того, что батарейка исправна, равна 0,98. Покупателю надо, чтобы и первая, и вторая батарейка были исправны: 0,98 · 0,98 = 0,9604.

Задача 9. На рок-фестивале выступают группы ― по одной от каждой из заявленных стран. Порядок выступления определяется жребием. Какова вероятность того, что группа из США будет выступать после группы из Канады и после группы из Китая? Результат округлите до сотых.

Решение: 

Тип вопроса: совмещение событий.

Общее количество выступающих на фестивале групп для ответа на вопрос неважно. Сколько бы их ни было, для указанных стран есть 6 способов взаимного расположения среди выступающих (КИТ — Китай, КАН = Канада):

… США, КАН, КИТ …

… США, КИТ, КАН …

… КИТ, США, КАН …

… КАН, США, КИТ …

… КАН, КИТ, США …

… КИТ, КАН, США …

США находится после Китая и Канады в двух последних случаях. Поэтому вероятность того, что группы случайным образом будут распределены именно так, равна:

формула 9

≈ 0,33.

Дополняющая вероятность

Задача 1. 

Автоматическая линия изготавливает батарейки. Вероятность того, что готовая батарейка неисправна, равна 0,02. Перед упаковкой каждая батарейка проходит систему контроля. Вероятность того, что система забракует неисправную батарейку, равна 0,97. Вероятность того, что система по ошибке забракует исправную батарейку, равна 0,05.

Найдите вероятность того, что случайно выбранная батарейка будет забракована.

Решение: 

Существуют 2 варианта, которые нам подходят:

Вариант А: батарейка забракована, она неисправна;

Вариант Б: батарейка забракована, она исправна.

Вероятность варианта А: 0,02 ∙ 0,97 = 0,0194;

Вероятность варианта Б: 0,05 ∙ 0,98 = 0,049;

Нас устроит либо первый, либо второй вариант: 0,0194 + 0,049 = 0,0684.

Задача 2. Две фабрики выпускают одинаковые стекла для автомобильных фар. Первая фабрика выпускает 60% этих стекол, вторая — 40%. Первая фабрика выпускает 3% бракованных стекол, а вторая — 5%. Найдите вероятность того, что случайно купленное в магазине стекло окажется бракованным.

Решение: 

Вероятность того, что стекло куплено на первой фабрике и оно бракованное: 0,6 · 0,03 = 0,018.

Вероятность того, что стекло куплено на второй фабрике и оно бракованное: 0,4 · 0,05 = 0,02.

Вероятность того, что случайно купленное в магазине стекло окажется бракованным, равна 0,018 + 0,02 = 0,038.

Задача 3. На фабрике керамической посуды 10% произведённых тарелок имеют дефект. При контроле качества продукции выявляется 80% дефектных тарелок. Остальные тарелки поступают в продажу. Найдите вероятность того, что случайно выбранная при покупке тарелка не имеет дефектов. Результат округлите до тысячных.

Решение: 

Предположим, у нас х тарелок изначально (ведь мы постоянно имеем дело с процентами, поэтому нам ничего не мешает оперировать конкретными величинами).

Тогда 0,1х — дефектные тарелки, а 0,9х — нормальные, которые поступят в магазин сразу. Из дефектных убирается 80%, то есть 0,08х, и остаётся 0,02х, которые тоже пойдут в магазин. Таким образом, общее количество тарелок на полках в магазине окажется: 0,9х + 0,02х = 0,92х. Из них нормальными будет 0,9х. Соответственно, по формуле вероятность будет 0,9х/0,92х ≈ 0,978.

Задача 4. По отзывам покупателей Игорь Игоревич оценил надёжность двух интернет-магазинов. Вероятность того, что нужный товар доставят из магазина А, равна 0,91. Вероятность того, что этот товар доставят из магазина Б, равна 0,89. Игорь Игоревич заказал товар сразу в обоих магазинах. Считая, что интернет-магазины работают независимо друг от друга, найдите вероятность того, что ни один магазин не доставит товар.

Решение. Вероятность того, что первый магазин не доставит товар, равна 1 − 0,91 = 0,09. Вероятность того, что второй магазин не доставит товар, равна 1 − 0,89 = 0,11. Вероятность происхождения двух этих событий одновременно равна произведению вероятностей каждого из них: 0,09 · 0,11 = 0,0099.

Задача 5. При изготовлении подшипников диаметром 70 мм вероятность того, что диаметр будет отличаться от заданного меньше чем на 0,01 мм, равна 0,961. Найдите вероятность того, что случайный подшипник будет иметь диаметр меньше чем 69,99 мм или больше чем 70,01 мм.

Решение: Нам дана вероятность события, при котором диаметр будет в пределах между 69,99 мм и 70,01 мм, и она равна 0,961. Вероятность всех остальных вариантов мы можем найти по принципу дополняющей вероятности: 1 − 0,961 = 0,039.

Задача 6. Вероятность того, что на тесте по истории учащийся верно решит больше 9 задач, равна 0,68. Вероятность того, что верно решит больше 8 задач, равна 0,78. Найдите вероятность того, что верно решит ровно 9 задач.

Решение: Вероятность того, что Т. верно решит более 8 задач, включает в себя вероятность решения ровно 9 задач. При этом, события, при которых О. решит больше 9 задач, нам не подходят. Следовательно, отняв от вероятности решения более 9 задач вероятность решения более 8 задач, мы и найдём вероятность решения только 9 задач: 0,78 – 0,68 = 0,1.

Задача 7. Из районного центра в деревню ежедневно ходит автобус. Вероятность того, что в понедельник в автобусе окажется меньше 21 пассажира, равна 0,88. Вероятность того, что окажется меньше 12 пассажиров, равна 0,66. Найдите вероятность того, что число пассажиров будет от 12 до 20.

Решение. Вероятность того, что в автобусе окажется меньше 21 пассажира, включает в себя вероятность, что в нём окажутся от 12 до 20 пассажиров. При этом события, при которых пассажиров будет меньше 12, нам не подходят. Следовательно, отняв от первой вероятности (менее 21) вторую вероятность (менее 12), мы и найдём вероятность того, что пассажиров будет от 12 до 20 : 0,88 – 0,66 = 0,22.

Задача 8. В Волшебной стране бывает два типа погоды: хорошая и отличная, причём погода, установившись утром, держится неизменной весь день. Известно, что с вероятностью 0,9 погода завтра будет такой же, как и сегодня. 10 апреля погода в Волшебной стране хорошая. Найдите вероятность того, что 13 апреля в Волшебной стране будет отличная погода.

Решение:

Задачу выполняют несколько вариантов («Х» — хорошая погода, «О» — отличная погода):

11 апреля 12 апреля 13 апреля Вероятность данного варианта
X – 0,9 X – 0,9 O – 0,1 0,9 ·0,9 ·0,1 = 0,081
X – 0,9 O – 0,1 O – 0,9 0,9 ·0,1 ·0,9 = 0,081
O – 0,1 O – 0,9 O – 0,9 0,1 ·0,9 ·0,9 = 0,081
O – 0,1 X – 0,1 O – 0,1 0,1 ·0,1 ·0,1 = 0,001

Вероятность происхождения какого-либо их этих 4-х вариантов равна сумме вероятностей каждого из вариантов: 0,081 + 0,081 + 0,081 + 0,001 = 0,244.

Задача 9. В Волшебной стране бывает два типа погоды: хорошая и отличная, причём погода, установившись утром, держится неизменной весь день. Известно, что с вероятностью 0,8 погода завтра будет такой же, как и сегодня. Сегодня 3 июля, погода в Волшебной стране хорошая. Найдите вероятность того, что 6 июля в Волшебной стране будет отличная погода.

Решение:

Задачу выполняют несколько вариантов («Х» ― хорошая погода, «О» ― отличная погода):

4 июля 5 июля 6 июля Вероятность данного варианта
X – 0,8 X – 0,8 O – 0,2 0,8 · 0,8 · 0,2 = 0,128
X – 0,8 O – 0,2 O – 0,8 0,8 · 0,2 · 0,8 = 0,128
O – 0,2 O − 0,8 O − 0,8 0,2 · 0,8 · 0,8 = 0,128
O – 0,2 X – 0,2 O – 0,2 0,2 · 0,2 · 0,2 = 0,008

Вероятность происхождения какого-либо их этих 4 ― х вариантов равна сумме вероятностей каждого из вариантов: 0,128 + 0,128 + 0,128 + 0,008 = 0,392.

Понравилась статья? Поделить с друзьями:
  • Задачи на вероятность с батарейками егэ
  • Задачи на вероятность монеты егэ
  • Задачи на вероятность егэ профильный формулы
  • Задачи на вероятность егэ профиль с решениями
  • Задачи на вероятность егэ батарейки